You are on page 1of 40

.

 VISIONIAS
™™™Ǥ˜‹•‹‘‹ƒ•Ǥ‹
ANSWERS & EXPLANATIONS
GENERAL STUDIES (P) TEST – 3818 (2023)

Q 1.A
x Direct Benefit Transfer
o The Direct Benefit Transfer program aims to bring transparency and terminate pilferage from the
distribution of funds sponsored by the Central Government of India. Benefits or subsidies are
immediately transferred to citizens living below the poverty level under DBT.
o The Office of the Controller General of Accounts’ Central Plan Scheme Monitoring System (CPSMS)
serves as a common platform for routing DBT. CPSMS can be used to prepare the beneficiary list,
digitally sign it, and process payments in the beneficiary’s bank accounts via the Aadhaar Payment
Bridge.
o Further, the scope of DBT has been further expanded to include in-kind transfers to beneficiaries as
well as transfers/honorariums given to various enablers of government schemes like ASHA, Anganwadi
workers, etc., and not limited to cash transfers to beneficiaries only.
o Note: Aadhar is not mandatory to access benefits under DBT.
o Validation of Beneficiary Account
9 These functions include features like application for the scheme by the beneficiary with a bank
account and Aadhaar details, review by scheme owners for eligibility of the beneficiary according
to scheme guidelines, beginning bank account/Aadhaar verification, payment initiation through
Fund Transfer Order, etc. These features make up a workflow-based framework for centrally
sponsored and state-linked schemes.
9 MGNREGA, PM-AWAS, PM-KISAN, DBT-PAHAL, etc. are some examples of such systems.
o Cash transfer: The government immediately forwards cash to each beneficiary under the Direct
Benefit Transfer cash transfer scheme. Following is a list of the numerous ways that money is
transferred from the government to beneficiaries:
9 The beneficiary may receive the amount directly.
9 The fund is transferred from the State Treasury Account to the beneficiary.
9 Through an Implementing Agency that has been assigned by the government.
9 The beneficiary gets the fund from the State or Central Government.
9 Some examples include the National Social Assistance Program (NSAP) and MGNREGA. Hence
option (a) is the correct answer.
o Direct Benefit Transfer in PDS
9 Recently, The Centre has decided to make the Direct Benefit Transfer (DBT) for the Public
Distribution System (PDS) optional for states as the pilot initiatives have failed to yield the desired
results.
o UJALA (Unnat Jyoti by Affordable LEDs for All) Scheme:
9 Launched in 2015 and initially labeled as the LED-based Domestic Efficient Lighting Programme
(DELP), it aims to promote the efficient usage of energy for all i.e., its consumption, savings, and
lighting.
9 Energy Efficiency Services Limited (EESL), a government company under the administrative
control of the Ministry of Power, has been designated as the implementing agency for this program.
9 Every domestic household having a metered connection from their respective Electricity
Distribution Company is eligible to get the LED bulbs under the Scheme. This scheme is not
covered under the DBT scheme.

1 www.visionias.in ©Vision IAS


.
 Q 2.D
x Post-independence, the Registration of Births and Death Act (RBD Act) was enacted in 1969 to promote
uniformity and comparability in the registration of Births and Deaths across the country and the compilation
 of vital statistics based thereon. With the enactment of the Act, registration of births, deaths and
stillbirths became mandatory in India. Hence statement 1 is correct.
x The Registrar General of India (RGI) at the Central Government level coordinates and unifies the
activities of registration throughout the country. Hence statement 2 is correct.
x However, implementation of the statute is vested with the State Governments. Hence statement 3 is
correct.
x The Act mandates the use of uniform birth and death reporting forms and certificates throughout the country.
x The scheme of Medical Certification of Cause of Death (MCCD) under the RBD Act, provides for causes
of death, a prerequisite to monitoring health trends of the population.

Q 3.C
x About Tele-MANAS: Tele Mental Health Assistance and Networking Across States (Tele-MANAS)
initiative was launched by the Union Ministry of Health & Family Welfare on the occasion of World
Mental Health Day under the National Tele Mental Health Programme (NTMHP) (announced in the
Union Budget 2022-23). Hence option (c) is the correct answer.
x Objectives: To further improve access to quality mental health counselling and care services in the country.
x It aims to provide free tele-mental health services all over the country. At least one Tele-MANAS Cell
in each State/UT.
x National Institute of Mental Health and Neurosciences (NIMHANS) is the nodal centre.
x Technical Support: International Institute of Information Technology-Bengaluru (IIITB), IIT-Bengaluru
and National Health Systems Resource Centre (NHRSC).
x A toll-free, 24/7 helpline number has been set up with the language of choice for availing services.
Hence statement 2 is correct.
x Integration: Tele-MANAS is also linked with other services like National tele-consultation service,
eSanjeevani, Ayushman Bharat Digital Mission, mental health professionals, Ayushman Bharat health and
wellness centres and emergency psychiatric facilities.

Q 4.B
x The National Archives of India is the custodian of the non-current records of the Government of India
and is holding them in trust for the use of the records creators and the users at large. It is the biggest
repository of the non-current records in South East Asia. Archives are the priceless documentary heritage
of any nation and as the premier archival institution in the country; the National Archives of India (NAI)
plays a key role in guiding and shaping the development of archives both at the national as well as
international level. In its unique position as the sole repository of the union government for public records
of the country.
x National Archives of India functions as an attached office of the Ministry of Culture entrusted with the
preservation of the documentary heritage of the nation. It is also the nodal agency for the implementation
of the Public Records Act, 1993 and public records. It has one regional office at Bhopal and three records
centres at Bhubaneswar, Jaipur and Puducherry. Hence statements 1 and 3 are not correct.
x The major activities of the National Archives of India include:
o making public records accessible to various Government agencies and research scholars; Hence
statement 2 is correct.
o an online search portal “www.abhilekh-patal.in” now facilitates to access records;
o rendering technical assistance to individuals and institutions in the field of conservation of records;
o the Department through its regular diploma courses are being conducted by the School of Archival
Studies imparts training in various disciplines of archival sciences.

Q 5.B
x The Rajiv Gandhi Scheme for Empowerment of Adolescent Boys – "Saksham" is launched by the
Ministry of State for Women and Child Development in 2014 for the all-round development of
adolescent boys (ABs) aged 11–18 years.
o The scheme aims to empower adolescent boys by providing them with the necessary education and
skills to become responsible citizens.
o It covers both school-going and out-of-school adolescent boys.
x The objectives of the Saksham scheme include gender sensitization, creating sensitised Ahimsa messengers
to address violence against women, enabling self-development and empowerment, addressing the physical,
2 www.visionias.in ©Vision IAS
.
 mental and emotional health needs of adolescent boys, promoting awareness about health, hygiene, nutrition
and Adolescent Reproductive & Sexual Health (ARSH) and family and child care, providing appropriate
information and vocational skills for ABs above 16 years through the National Skill Development
 Programme (NSDP), providing necessary life skills education, providing information and guidance about
existing public services, and channelizing the energies of ABs for the development of the nation.
x Hence option (b) is the correct answer.

Q 6.B
x Ishan Vikas - It aims to bring academic exposure for North- Eastern Students. Hence pair 1 is
correctly matched.
o The programme has been launched with a plan to bring selected college and school students from the
north-eastern states into close contact with IITs, NITs and IISERs during their vacation periods for
academic exposure.
x Ishan Uday - for Students of the North-East Region The UGC launched a special scholarship scheme
for students of north-east region, Ishan Uday, from the academic session 2014-15. Hence pair 2 is not
correctly matched.
o The Scheme envisages grant of 10,000 scholarships to students from the north-east region whose
parental income is below ₹ 4.5 lakh per annum for undergraduate level in college/universities in India.
The sale of scholarship is ₹ 54,000 per annum for general degree programme and ₹ 78,000 p.a. for
technical and professional programme.
x Pragati (scholarship for girl students) is a scheme of AICTE aimed at providing assistance for
advancement of girls, participation in technical education. Hence pair 3 is correctly matched.
o This is an attempt to give every young woman the opportunity to further her education and prepare for
a successful future. Salient features of the scheme include a number of scholarships per annum:₹ 4,000
’One Girl’ per family, where the family income is less than ₹ 6 lakh per annum.
o The candidates will be selected on merit after the qualifying examination to pursue technical education
from amongst such candidates. Amount of scholarship: tuition fee of ₹ 30,000/- or at actual, whichever
is less and ₹ 2,000/- per month for 10 months as incidentals each year. 15 per cent for SC, 7.5 per cent
ST and 27 percent for OBC candidate/applicant are reserved.
o Out of the total number of scholarships in each scheme, 50 per cent of them are available at each
degree/diploma level and also transferrable in the event of non-availability of eligible applicant.
Q 7.C
x Pradhan Mantri Urja Ganga project is also known as Jagdishpur – Haldia & Bokaro – Dhamra Pipeline
Project (JHBDPL). Hence option (c) is the correct answer.
x It is a gas pipeline project originating at Jagdishpur (Uttar Pradesh), aims to provide cooking gas to
people. It was launched in 2016 in Varanasi and later extended to people in the states of Bihar, West
Bengal, Odisha and Jharkhand.
x The government is taking the initiative to connect the eastern states with the national gas grid.
x The total length of the pipeline under the Pradhan Mantri Urja Ganga Project is approximately 3,384 km,
out of which 766 km of pipeline is in Odisha state and the balance 2,618 km is in the states of Uttar Pradesh,
Bihar, Jharkhand, West Bengal & Assam.
x The project will not just supply CNG to automobiles and cooking gas to household kitchens in cities along
the route, but also to industries to meet their feedstock or fuel requirement.
x The seven main station cities include Varanasi, Patna, Bokaro, Jamshedpur, Kolkata, Ranchi, Bhubaneswar
and Cuttack as the major beneficiaries of the project.
x GAIL is executing Jagdishpur-Haldia-Bokaro-Dhamra Pipeline (JHBDPL) of length 2,655 km and Barauni-
Guwahati Pipeline of length 729 km under Pradhan Mantri Urja Ganga Project to connect Eastern India.
Total length of pipeline under Pradhan Mantri Urja Ganga Project is approx. 3,384 km, out of which 766
km of pipeline is in Odisha state and the balance 2,618 km is in the states of Uttar Pradesh, Bihar, Jharkhand,
West Bengal & Assam. At present the pipeline has been commissioned upto Barauni in Bihar.

Q 8.C
x Seventh Plan (1985–1990) aimed at the rapid growth in food grain production, increased employment
opportunities, and productivity within the framework of basic tenets of planning namely growth,
modernization, self-reliance, and social justice.
x Foodgrain production increased by 3.23 percent as compared to the long-term growth rate of 2.63 percent
between 1967-68 and 1988-89.
x To reduce unemployment and the incidence of poverty, the Jawahar Rozgar Yojana was launched in
1989 in addition to the existing programs.
3 www.visionias.in ©Vision IAS
.
 x During the plan period, GDP grew at an average of 5.8 percent exceeding the targeted growth rate by 0.8
percent.
x Hence, option (c) is the correct answer.

Q 9.A
x Antyodaya Anna Yojana (AAY) was launched on 25th December 2000. It was launched to provide
food grains at a highly subsidized rate of ₹3 per kg for rice, ₹2 per kg of wheat, and ₹1 per kg for
coarse grains under TPDS (Targeted Public Distribution System). Hence statement 2 is correct.
x Initially, every BPL family was allowed 25 kg of grains per month. Since 2002, the entitlement for
each family has been increased to 35 kg per month. These families are BPL cardholders. Hence
statement 1 is correct.
x Antyodaya Anna Yojana: Intended Beneficiaries
o Landless Agricultural Labourers, Marginal Farmers, Rural Artisans, etc.
o Households with Widows, or terminally ill patients as the head of the family.
o Widows or terminally ill patients living alone.
o BPL families of HIV-positive persons.
o Primitive tribal households.
x The distribution cost, including a margin to dealers and retailers as well as the transportation cost, is
borne by states/UTs. Hence statement 3 is not correct.

Q 10.C
x IND-SAT: The Ministry of Education conducted the first ever Indian Scholastic Assessment (Ind-SAT)
Test 2020 under its Study in India programme in 2020.
o Ind- SAT is an exam for grant of scholarships and admissions to foreign students for studying in
select Indian universities under the Study in India programme. Hence pair 1 is not correctly
matched.
x Study in India: This is a programme for attracting foreign students to study in the best higher
education institutions in India. Hence pair 2 is correctly matched.
o Currently, 116 institutions have been selected for the programme. While the programme is open to all
the countries in the world, the focus is on 42 African and Asian countries. Students apply to institutions
of their choice through the SII portal and are selected on the basis of their marks in their school final
qualifying exams. Institutions offer merit-based fee waivers ranging from 25 to 100 per cent.
x Growing Aptitude in Numerical Innovations and Training (GANIT) week in schools affiliated with
CBSE was organised to commemorate the birth anniversary of Srinivasa Ramanujan and to actively
promote the interest of students in mathematics. Hence pair 3 is correctly matched.

Q 11.A
x Falkenmark indicator is used for water stress that expresses the level of water scarcity in a certain region
as the amount of renewable freshwater that is available for each person each year. Hence, statement
1 is correct.
x Water availability below 1,000 cubic metres is considered a water scarcity condition, whereas, annual
per capita water availability of less than 1700 cubic metres is considered as a water-stressed
condition. Hence, statement 2 is correct.
x Per capita, water availability in India has reduced to 1,500 cubic metres due to increased population and
water usage. It is further projected to reduce to 1,340 and 1,140 in 2025 and 2050 respectively. Hence,
statement 3 is not correct.

Q 12.C
x Jurisdiction and Seat of High Courts
o Each high court has powers of superintendence over all courts and tribunals within its jurisdiction. It
can call for returns from such courts, make and issue general rules and prescribed forms to regulate
their practices and proceedings and determine the manner and form in which book entries and accounts
shall be kept.
4 www.visionias.in ©Vision IAS
.


x Hence only pairs 1 and 4 are correctly matched.


5 www.visionias.in ©Vision IAS
.
 Q 13.D
x The PLI scheme was designed to increase domestic manufacturing capability while also increasing import
substitution and employment generation.
 x In Budget 2022-23, the government has set aside Rs 1.97 lakh crore for PLI schemes in various sectors,
with an additional allocation of Rs 19,500 crore for PLI for solar PV modules.
x The scheme, which was launched in March 2020, initially targeted three industries: Medical Devices,
Mobile and allied Component Manufacturing Electrical Component Manufacturing.
x The Scheme provides the following Incentives: The incentives, computed on incremental sales, range
from as low as 1% for electronics and technology products to as high as 20% for the manufacturing of
critical key starting drugs and certain drug intermediaries.
x In some sectors, such as advanced chemistry cell batteries, textile products, and the drone industry, the
incentive will be computed based on sales, performance, and local value addition over a five-year period.
x The government has so far announced PLI schemes for 14 sectors, including automobiles and auto
components, electronics and IT hardware, telecom, solar modules, pharmaceuticals, metals and mining,
textiles and clothing, white goods, drones, and advanced chemical cell batteries.
x Objectives
o This scheme was implemented by the government in order to reduce India's dependence on China and
other foreign countries.
o It promotes labor-intensive sectors and aims to boost India's employment ratio.
o This scheme aims to reduce import bills and boost domestic production.
o PLI Yojana, on the other hand, encourages domestic enterprises to expand their production units and
invites foreign companies to set up their units in India.
x Sectors Under PLI SchemeThe program is specifically made to increase domestic manufacturing in
strategic and sunrise industries, lower import costs and cheaper imports, improve the cost competitiveness
of domestically produced goods, increase domestic capacity, and promote exports. Sectors under PLI
Scheme are:

x Hence option (d) is the correct answer.

Q 14.B
x SkillsBuild is a digital platform developed by IBM that enables holistic learning and aligns it with the
Skills India initiative of Government of India. India is the 4th country where this Skills Build platform
has been launched, on November 4, 2019, in alliance with the Directorate General of Training (DGT),
6 www.visionias.in ©Vision IAS
.
 Ministry of Skill Development and Entrepreneurship (MSDE), after being launched in the UK, Germany
and France. Hence statement 1 is not correct.
x Skill Build focuses on personalised coaching and experiential learning and will help develop the skills
 required to join the workforce in the "New Collar" roles.
x It also brings untapped talent to the fore, opening roles to individuals often excluded from the labour market
due to disadvantaged backgrounds, or because of lack of knowledge, skill, or experience that has seen them
become under-employed or unemployed. Hence statement 2 is correct.
x As part of the programme, a two-year advanced diploma in Information Technology (IT), Networking and
Cloud computing will be offered at the Industrial Training Institutes (ITI) and the National Skill Training
Institutes (NSTIs). The platform will also be extended to train ITI and NSTI faculty in basic Artificial
Intelligence (AI) skills.
x The platform also provides personal assessment for cognitive capabilities and personality to youth and
students. They will learn the basics about digital technologies as well as professional skills like resume
writing, problem solving and communication. They will also receive recommendations on role-based
education for specific jobs that include technical and professional learning.

Q 15.C
x Recent context: Agriculture Infrastructure Fund (AIF) crosses Rs.30,000 crore mark of capital mobilisation
for projects in agriculture sector for creation of post-harvest management infrastructure and community
farming assets.
x AIF is a Central Sector Scheme to mobilize a medium - long term debt financing facility for investment in
viable projects relating to post- harvest management Infrastructure and community farming assets through
incentives and financial support.
o AIF is a financing facility launched in July 2020.
o It aims to provide all-around financial support to the farmers, agri-entrepreneurs, farmer groups like
Farmer Producer Organisations (FPOs), Self Help Groups, Joint Liability Groups (JLGs) etc. and many
others to create post-harvest management infrastructure and build community farming assets throughout
the country. Hence statement 1 is correct.
x Features:
o Credit guarantee coverage will be available for eligible borrowers from this financing facility under
Credit Guarantee Fund Trust for Micro and Small Enterprises (CGTMSE) scheme for loans up to ₹ 2
crore. The fee for this coverage will be paid by the Government. In case of FPOs the credit guarantee
may be availed from the facility created under FPO promotion scheme of DA&FW. Hence statement
2 is correct.
o All loans under this financing facility will have interest subvention of 3% per annum up to a limit of ₹
2 crore. This subvention will be available for a maximum period of 7 years. In case of loans beyond ₹
2 crore, then interest subvention will be limited up to ₹ 2 crore.
o AIF is helping in reducing post-harvest losses by creating and modernising agriculture infrastructure,
which includes primary processing centres for vegetables, hi-tech hubs for rental of agricultural
machinery.
x Management:
o The fund will be managed and monitored through an online Management Information System (MIS)
platform. It will enable all the qualified entities to apply for loans under the Fund.
9 The National, State and District level monitoring committees will be set up to ensure real-time
monitoring and effective feed-back.
Q 16.D
x Through sustained programme of investment and greater thrust on application of modern technologies, it
has been possible to raise the All India production of coal at 716.08 million tonnes in 2020-21 (Provisional).
The all India Production of coal during 2021-22 were 778.19 MT (Provisional) with a positive growth of
8.67%.
x Coal India Limited (CIL) and its subsidiaries accounted for 596.221 million tonnes during 2020-21 as
compared to a production of 602.129 million tonnes in 2019-20 showing a negative growth of 0.98%. Coal
production of CIL during 2021-22 were 622.634 MT(Provisional) with a positive growth of 4.43%
(approximately 80% of the total coal production in the country). Hence statement 3 is correct.
x India can save up to $19.5 billion a year if it goes with its plan to add 76 GW of utility-scale solar and wind
power by 2025, new research from Global Energy Monitor has stated. India announced plans to source half
of its electricity needs from non-fossil fuel sources by 2030. India’s current energy mix is still dominated
by coal. At present, coal accounts for 44 per cent of India’s primary energy sources and 70 per cent of its
power generation. Hence statement 2 is correct.
7 www.visionias.in ©Vision IAS
.


x Hence statement 1 is correct.

Q 17.C
x Union Minister for Social Justice and Empowerment Dr Virendra Kumar launched an online portal
TAPAS (Training for Augmenting Productivity and Services), developed by the National Institute of
Social Defence.
x The idea of TAPAS was conceptualised at a time when exploring the online medium for work and education
had become imperative due to the outbreak of Covid 19 pandemic. What fuelled it further was the Ministry’s
vision of ensuring maximum participation of stakeholders and volunteers working in the field of social
defence for better training and capacity building.
x TAPAS is the initiative of the National Institute of Social Defence (NISD), Ministry of Social Justice
and Empowerment, to provide access to lectures by subject experts, study material and more, but in
a manner that it supplements the physical classroom without compromising on the quality of
teaching. Hence, option (c) is correct.
x The main objective of introducing the course modules is to impart training and enhance the knowledge and
skills for the capacity building of the participants. It can be taken up by anyone who wishes to enhance his
or her knowledge of the topics and there is no fee for joining.
x The five basic courses are on Drug (Substance) Abuse Prevention, Geriatric/Elderly Care, Care and
Management of Dementia, Transgender Issues and on the comprehensive course on Social Defence Issues.
x TAPAS is a standard MOOC (Massive Open Online Course) platform with course material such as filmed
lectures and e-study material. It also includes discussion forums to support and encourage interactions
among students and course coordinators.
x The platform has been made with a quadrant approach. There will be a virtual class where an expert or a
resource person will give a lecture about the subject with the help of animated infographics and Powerpoint
presentations. The participants will be provided with informative material like articles, case studies and
other study material on the related topic, which can be downloaded for future reference. After completing
each module, students can assess their understanding of the subject with the help of a multiple-choice quiz.
There is also a discussion forum on the portal on which they can raise their doubts with the course
coordinator.

Q 18.D
x The Ministry of Skill Development and Entrepreneurship (MSDE) launched the Pradhan Mantri
YUVA Yojana (PM-YUVA) or Yuva Udyamita Vikas Abhiyan on November 9, 2016, marking the
second Foundation Day of the Ministry. It is the flagship scheme of MSDE on entrepreneurship education
and training.
x The scheme aims to promote entrepreneurship as an alternative career choice and enable sustained long-
term mentoring support through the journey to potential and early-stage entrepreneurs by imparting
entrepreneurship education and mentoring support to trainees/beneficiaries from the skilling ecosystem.
x The PM-YUVA Yojana is aligned with the Aatmanirbhar, or self-reliant India as it would enable the
promotion of entrepreneurship and entrepreneurial ventures in the country. The scheme provides easy
access to information and a mentor network, credit, an incubator and accelerator, and advocacy to create a
pathway for the youth.
x Hence option (d) is the correct answer.

Q 19.A
x Mission Purvodaya is a flagship initiative of the Ministry of Steel, Petroleum and Natural Gas. It was
launched in 2020 for the accelerated development of eastern India through the establishment of
an integrated steel hub in Kolkata, West Bengal.
8 www.visionias.in ©Vision IAS
.
 x It will drive best-in-class capacity creation, augment value addition and boost competitiveness through the
setting up of Greenfield steel plants, clusters, capital goods, and requisite logistics infrastructure.
x Hence, option (a) is the correct answer.

Q 20.C
x Infrastructure Finance Secretariat (IFS):
o The Department of Economic Affairs (DEA) in the Ministry of Finance had two divisions (the
Infrastructure Policy and Planning Division, and the Infrastructure Support and Development Division)
overseeing policy matters relating to the infrastructure sector.
o These two divisions are now being moved to a new set-up called the Infrastructure Finance Secretariat
(IFS). The IFS will have three sections: Infrastructure financing, Sectoral studies, and Capacity
building.
x Financial Stability Board (FSB): FSB is an international body that monitors and makes recommendations
about the global financial system. It was established in April 2009 after the G20 Summit in London as
the successor to the Financial Stability Forum.
o Mandate: To coordinate at the international level the work of national financial authorities and
international standard-setting bodies and to develop and promote the implementation of effective
regulatory, supervisory, and other financial sector policies.
o The FSB, working through its members, seeks to strengthen financial systems and increase the stability
of international financial markets.
o Headquarters: Basel, Switzerland.
o The board includes all G20 major economies. The FSB consists of 68 member institutions.
x Financial Stability and Development Council (FSDC)
o The Financial Stability and Development Council (FSDC) was constituted by an Executive Order of
the Union Government as a non-statutory apex body under the Ministry of Finance in 2010. Hence
option (c) is the correct answer.
o FSDC – Council Members
9 The Finance Minister is the Chairman of the FSDC.
9 Members of FSDC include Heads of the Financial Sector Regulators listed below:
9 Reserve Bank of India (RBI)
9 Insurance Regulatory and Development Authority (IRDA)
9 Securities and Exchange Board of India (SEBI)
9 Pension Fund Regulatory and Development Authority (PFRDA)
9 Other members are Finance Secretary, Chief Economics Advisor, and Secretary of the Department
of Financial Services.

Q 21.D
x The Petroleum and Natural Gas Regulatory Board (PNGRB) was constituted under The Petroleum and
Natural Gas Regulatory Board Act, 2006. Hence statement 1 is not correct.
x The Act provide for the establishment of Petroleum and Natural Gas Regulatory Board to protect the
interests of consumers and entities engaged in specified activities relating to petroleum, petroleum products
and natural gas and to promote competitive markets and for matters connected therewith or incidental
thereto.
x Further as enshrined in the act, the board has also been mandated to regulate the refining, processing,
storage, transportation, distribution, marketing and sale of petroleum, petroleum products and natural
gas excluding production of crude oil and natural gas so as and to ensure uninterrupted and adequate
supply of petroleum, petroleum products and natural gas in all parts of the country. Hence statement 2
is not correct.

Q 22.D
x Ministry of Home Affairs (Grih Mantralaya)
o Department of Internal Security (Aantarik Suraksha Vibhag)
o Department of States (Rajya Vibhag). Hence pair 1 is not correctly matched.
o Department of Official Language (Raj Bhasha Vibhag)
o Department of Home (Grih Vibhag)
o Department of Jammu Kashmir and Ladakh Affairs (Jammu Kashmir aur Ladakh Vibhag)
o Department of Border Management (Seema Prabandhan Vibhag). Hence pair 3 is not correctly
matched.

9 www.visionias.in ©Vision IAS


.
 x Ministry of Heavy Industries and Public Enterprises (Bhari Udyog aur Lok Udyam Mantralaya)
o Department of Heavy Industries (Bhari Udyog Vibhag)
o Department of Public Enterprises (Lok Udyam Vibhag). Hence pair 2 is not correctly matched.
 x Ministry of Personnel, Public Grievances and Pensions (Karmik, Lok Shikayat tatha Pension
Mantralaya)
o Department of Personnel and Training (Karmik aur Prashikshan Vibhag)
o Department of Administrative Reforms and Public Grievances (Prashasnik Sudhar aur Lok Shikayat
Vibhag)
o Department of Pensions and Pensioners’ Welfare (Pension aur Pension Bhogi Kalyan Vibhag).
Hence pair 4 is not correctly matched.
x Ministry of Defence (Raksha Mantralaya)
o Department of Defence (Raksha Vibhag)
o Department of Military Affairs (Sainya Karya Vibhag)
o Department of Defence Production (Raksha Utpadan Vibhag) (iv) Department of Defence Research
and Development (Raksha Anusandhan tatha Vikas Vibhag)
o Department of Ex-Servicemen Welfare (Poorva Senani Kalyan Vibhag)
x Ministry of Commerce and Industry (Vanijya aur Udyog Mantralaya)
o Department of Commerce (Vanijya Vibhag)
o Department for Promotion of Industry and Internal Trade (Udyog Samvardhan aur Antarik Vyapar
Vibhag)
x Ministry of Labour and Employment (Shram aur Rozgar Mantralaya). It has no department under it.
x Ministry of Parliamentary Affairs (Sansadiya Karya Mantralaya). It has no department under it.
Q 23.C
x Pradhan Mantri Garib Kalyan Anna Yojana (PM-GKAY) is a scheme as part of Atmanirbhar Bharat
to supply free food grains to migrants and poor.
o Phase-I and Phase-II of this scheme was operational from April to June, 2020 and July to November,
2020 respectively;
o Phase III of the scheme was operational from May to June, 2021;
o Phase-IV of the scheme during July-November, 2021 and
o Phase V from December 2021 till March, 2022.
x PMGKAY provides 5 kg of foodgrains to each family holding a ration card free of cost and the 5 kg
of subsidized foodgrain already offered through the Public Distribution System (PDS). This is in
addition to the subsidized (Rs 2-3 per kg) ration provided under the National Food Security Act (NFSA) to
families covered under the Public Distribution System (PDS). The food grain and the amount may be
variable.
o Wheat has been allocated to 6 States/UTs, - Punjab, Haryana, Rajasthan, Chandigarh, Delhi, and
Gujarat, and rice has been provided to the remaining States/UTs
x Eligibility:
o Families belonging to Antyodaya Anna Yojana (AAY) and Priority Households (PHH) categories
will be eligible for the scheme. Hence, statement 1 is not correct.
9 PHH are to be identified by State Governments/Union Territory Administrations as per criteria
evolved by them. AAY families are to be identified by States/UTs as per the criteria prescribed by
the Central Government:
o Households headed by widows or terminally ill persons or disabled persons or persons aged 60
years or more with no assured means of subsistence or societal support.
9 Widows or terminally ill persons or disabled persons or persons aged 60 years or more or single
women or single men with no family or societal support or assured means of subsistence.
o All primitive tribal households.
o Landless agriculture labourers, marginal farmers, rural artisans/craftsmen such as potters, tanners,
weavers, blacksmiths, carpenters, slum dwellers, and persons earning their livelihood on daily basis in
the informal sector like porters, coolies, rickshaw pullers, hand cart pullers, fruit and flower sellers,
snake charmers, rag pickers, cobblers, destitutes and other similar categories in both rural and urban
areas.
o All eligible Below Poverty Line families of HIV positive persons
x The PMGKAY is subsuming two subsidy schemes:
o Food Subsidy to Food Corporation of India (FCI).
o Food Subsidy for Decentralized Procurement (DCP).
x However, there will be no changes in the issue prices of food grains supplied to States for other welfare
programmes like mid-day meals (MDMs). Hence, statement 2 is not correct.
 10 www.visionias.in ©Vision IAS
.
 Q 24.A
x India is the world's fifth-largest economy by nominal GDP and is one of the fastest-growing economies
globally. Efficient logistics is the bedrock for a growing economy like India. The reduction in logistics costs
 could be a key enabler in enhancing the competitiveness of all sectors of the economy.
x Recognising the need to instill a logistics and supply chain mindset across States/UTs, the Ministry of
Commerce and Industry (MoCI) launched a study, "Logistics Ease Across Different States
(LEADS)" in 2018, with the main objective of ranking States and UTs on the efficiency of their
logistics ecosystems.
x The LEADS survey assesses the viewpoints of various users and stakeholders across the value chain
(Shippers, Terminal Infrastructure Service Providers, Logistics Service Providers, Transporters and
Government agencies) to understand the ‘enablers’ and ‘impediments’ to the logistics ecosystem in the
country. The annual survey processes the data received from stakeholders (perception data) and States/ UTs
(objective data) and ranks the logistics ecosystem of each State/ UT using a statistical model.
x Andhra Pradesh, Maharashtra, Gujarat and 12 other states and Union territories (UTs) have emerged as
"achievers" in ensuring the ease of logistics services. The ranking is based on their performance in the
logistics index chart 2022, which is part of the Logistics Ease Across Different States (LEADS) 2022 report.
x Kerala, Madhya Pradesh, Rajasthan, Puducherry, Sikkim and Tripura are the other States categorised as
"fast movers" in the ranking.
x The 15 States and UTs ranked in the "aspirers" category include Bihar, Chhattisgarh, Goa, Mizoram,
Andaman and Nicobar Islands, Lakshadweep, Ladakh, Nagaland, Jammu and Kashmir, and Arunachal
Pradesh.
x The other States and UTs that are in the “achievers” category include Chandigarh, Delhi, Haryana, Himachal
Pradesh, Karnataka, Maharashtra, Odisha, Punjab, Tamil Nadu, Telangana, Uttar Pradesh and Uttarakhand.
x The index aims at enhancing the focus on improving logistics performance across states, which is essential
for improving the country's trade and reducing transaction costs.
x The LEADS 2022 survey further propels the transformative approach of PM-Gati Shakti. It helps
identify issues and bottlenecks that need immediate action and can help synergize the supply chain, break
down silos, and improve the logistics efficiency of the country.
x Hence option (a) is the correct answer.

Q 25.C
x The National Repository of Information for Women (NARI) portal is an initiative of the Ministry of
Women and Child Development aimed at providing easy access to information on government
schemes and initiatives for women.
x The government, both central and state, has implemented a number of schemes and legislations for women
to provide them with equal rights, economic opportunities, social support, legal aid, housing, and more.
However, lack of awareness of these provisions and their scattered nature on different websites/portals make
it difficult for women to access their benefits.
x To address this issue, the NARI portal has summarized over 350 government schemes and other important
information for the benefit of women, with more being added every day. The schemes are divided into seven
different categories, namely education, health, employment, housing and shelter, addressing violence,
decision making, and social support.
x The portal provides links to the ministries, departments, and autonomous bodies offering these schemes, as
well as easy access to online applications and grievance redressal.
x Women can also find information on issues affecting their lives, including tips on good nutrition,
suggestions for health check-ups, information on major diseases, tips for job search and interview,
investment and savings advice, information on crimes against women and reporting procedures, contacts of
legal aid cells, simplified adoption procedures, and much more.
x It endows women with the power of information to build their life skills and facilitates them in taking full
advantage of the services provided by the government for them.
x Hence option (c) is the correct answer.

Q 26.C
About National Curriculum Framework (NCF) for Foundational Stage
x NCF Development: As per the National Education Policy (NEP), 2020, the following four NCFs will be
developed:
o National Curriculum Framework for Early Childhood Care and Education (NCFECCE).
o National Curriculum Framework for School Education (NCFSE).

 11 www.visionias.in ©Vision IAS


.
 o National Curriculum Framework for Teacher Education (NCFTE).
o National Curriculum Framework for Adult Education (NCFAE).
x Key highlights of NCF for the Foundational Stage:
 o Importance to mother tongue: Students should be taught exclusively in their mother tongue until
the age of eight. Hence statement 1 is correct.
o Less burden on children: For ages three to six, there should not be any prescribed textbooks for the
children, instead NCF recommended simple worksheets for curricular goals and pedagogical needs.
x The framework has listed the 'panchakosha' concept
for children's education. The Panchakosha description
is in the Taittiriya Upanishad. Hence statement 2 is
correct.
x It comprises:
o Physical development (sharirik vikas),
o Development of life energy (pranik vikas),
o Emotional and mental development (manasik vikas),
o Intellectual development (bauddhik vikas).
o Spiritual development (chaitsik vikas).

Q 27.C
x Poshan Abhiyaan: Poshan Abhiyaan is a flagship programme of the Ministry of Women and Child
Development which ensures convergence with various programmes i.e., Anganwadi Services, Pradhan
Mantri Matru Vandana Yojana (PMMVY), Scheme for Adolescent Girls (SAG) of MWCD Janani Suraksha
Yojana (JSY), National Health Mission (NHM), Swachh Bharat Mission and Public Distribution System
(PDS). It focuses to lay emphasis on the first 1,000 days of the child, which includes the nine months
of pregnancy, six months of exclusive breastfeeding and the period from 6 months to 2 years to ensure
focused interventions on addressing undernutrition, besides increasing the birth weight. Hence statement
2 is correct.
x It will help reduce both Infant Mortality Rate (IMR) and Maternal Mortality Rate (MMR). An additional
year of sustained intervention (till the age of 3 years) would ensure that the gains of the first 1,000 days are
consolidated. Attention is also given to children in the age group of 3-6 years for their overall development
through the platform of the Anganwadi Services. It aims to ensure holistic development and adequate
nutrition for pregnant women, mothers, and children. Hence statement 1 is correct.
x Implementation status reports of the Abhiyaan are submitted by NITI Aayog every six months. The
implementation of POSHAN Abhiyaan is carried out through the Technical Support Unit (TSU) established
at NITI Aayog, which will also provide research, policy, and technical support for the program. Hence
statement 3 is not correct.

 12 www.visionias.in ©Vision IAS


.
 Q 28.A
x 'Raja Harishchandra' was the first Indian movie released in 1913. It was a silent film directed and
produced by Dadasaheb Phalke. Hence, statement 1 is correct.
 x Legendary actress Asha Parekh received the prestigious Dada Saheb Phalke Award at the 68th National
Film Awards in 2022 for her exemplary lifetime contribution to Indian Cinema. This is the highest
recognition in the field of Indian cinema. Hence, statement 2 is not correct.

Q 29.D
x NI-KSHAY-(Ni=End, Kshay=TB) is the web-enabled patient management system for TB
control under the National Tuberculosis Elimination Programme (NTEP).
x It is developed and maintained by the Central TB Division (CTD), Ministry of Health and Family
Welfare, Government of India, in collaboration with the National Informatics Centre (NIC), and the
World Health Organization Country Office for India.
x It is vividly used by health functionaries at various levels across the country both in the public and private
sector, to register cases under their care, order various types of tests from Labs across the country, record
treatment details, monitor treatment adherence, and transfer cases between care providers.
x It also functions as the National TB Surveillance System and enables reporting of various surveillance
data to the Government of India.
x TB:
o It is caused by a bacterium called Mycobacterium tuberculosis, belonging to the Mycobacteriaceae
family.
o In humans, TB most commonly affects the lungs (pulmonary TB), but it can also affect other organs
(extra-pulmonary TB).
o It is an ancient disease and has been documented to have existed in Egypt as early as 3000 BC.
o Transmission: It is a communicable disease spread from person to person through the air.
o Symptoms: Cough with sputum and blood at times, chest pains, weakness, weight loss, fever, and
night sweats.
o Treatment: It is treatable and curable with Anti-TB medicine.
x Hence option (d) is the correct answer.

Q 30.C
x The Ministry of Consumer Affairs, Food & Public Distribution has launched a central sector scheme-
Integrated Management of Public Distribution System (IMPDS). Hence statement 1 is correct.
x The primary goal of this scheme is to implement nationwide portability of ration cards issued by
States/UTs under the National Food Security Act, 2013 (NFSA) for anywhere distribution of
subsidized food grains to ration card holders in India. Hence statement 2 is correct.
x The system is now known as the 'One Nation One Ration Card (ONORC) Plan.'
x The ONORC plan allows NFSA beneficiaries, particularly migrant beneficiaries, to obtain their entitled
foodgrains from any ePoS-enabled Fair Price Shop (FPS) in the country by using their same/existing NFSA
ration card after biometric/Aadhaar authentication on an electronic Point of Sale (ePoS) device.
x So far, the ONORC plan has been activated in 35 states/UTs, covering approximately 77 million
beneficiaries in the country, who now have the option of accessing the Targeted Public Distribution System
(TPDS) and their entitled foodgrains from any nearby FPS in these 35 states/UT.
x The scheme is being implemented with the technical assistance of the National Informatics Centre (NIC),
which is also assisting the States/UTs in the upgrade of ePoS software, integration with the central IM-PDS
and Annavitran portals, availability of ration cards/beneficiaries' data in the Central Repository.

Q 31.C
x Indradhanush Gas Grid Limited (IGGL) is a Joint Venture of IOCL, ONGG, GAIL, OIL and NRL. IGGL
was incorporated on the 10th of August 2018, towards implementing the Hydrocarbon Vision 2030 for
North-East India released by Ministry of Petroleum & Natural Gas in 2016. Hence statement 1 is correct.
x Hydrocarbon Vision 2030 envisages NorthEast India economic development by leveraging the region's
Hydrocarbon potential, enhancing access to clean fuel and accelerating the growth of entire North East
India.
x The Gas Grid Project:
o As per the plan on the Vision Document, The Gas Grid Project connects eight North- Eastern States,;
Assam, Arunachal Pradesh, Meghalaya, Manipur, Mizoram, Nagaland, Tripura and Sikkim, to the
National Gas Grid through Barauni-Guwahati Gas Pipeline. From Guwahati, the pipeline extends to
Numaligarh, Dimapur, Kohima and Imphal in one direction; Shillong, Silchar, Aizawl and Agartala in
 13 www.visionias.in ©Vision IAS
.
 the second direction and to Itanagar in the third direction. Gangtok gets connected to Siliguri from the
gas pipeline coming from Barauni to Guwahati.
o The natural gas grid in North east is planned to connect with upcoming Barauni-Guwahati natural gas
 pipeline as a part of Urja-Ganga scheme. The grid would also connect to sustainable and viable gas
sources in North-east. Hence statement 2 is correct.

Q 32.B
x Tele-density denotes the number of telephones per 100 population. It is an important indicator of
telecom penetration. The overall teledensity in India was 86.89 percent at the end of November 2021.
x Himachal Pradesh (142.49 percent) had the highest teledensity followed by Kerala (127.98 percent),
Punjab (122.97 percent), and Tamil Nadu (107.08 percent).
x Hence, option (b) is the correct answer.

Q 33.B
x Common Equity Tier 1 (CET1)
o Common Equity Tier 1 (CET1) is a component of Tier 1 capital that is primarily common stock held
by a bank or other financial institution.
o CET1 is a capital measure that was introduced in 2014 as a precautionary way to protect the economy
from a financial crisis, largely in the context of the European banking system.
o A bank’s capital structure consists of several tiers. These include:
9 Tier 1 Capital: Known as going concerned or core capital, Tier 1 is used to fund a financial
institution's business activities. It includes Common Equity Tier 1 (CET1) capital and Additional
Tier 1 (AT1) capital.
9 Tier 2 Capital: This is often called gone concern or supplementary capital. This category is made
up of things like hybrid capital instruments and subordinated term debt.
9 Tier 3 Capital: This type of capital includes market risk, commodities risk, and foreign currency
risk and is the lowest quality of the three.
x Capital to Risk (Weighted) Assets Ratio (CRAR)
o It is a bank’s capital proportion to its current obligations and risk-weighted assets. Central banks and
bank regulators decide to stop commercial banks from using excessive leverage and going bankrupt.
o CRAR in Banking
9 To guarantee that banks have sufficient protection against loss before they become bankrupt and
lose depositor money, minimum capital ratios (CARs) are crucial.
9 By lowering the risk of bank insolvency, CRARs promote the effectiveness and sustainability of a
country’s financial system.
9 A bank is widely seen as secure and much more likely to meet its financial commitments if it has a
high Capital Adequacy Ratio.
9 Account holders can lose their savings if a bank experiences a loss that surpasses the capital it has
on hand since creditor funds are prioritized over the bank’s capital throughout the winding-up
process. Consequently, the greater the depositors’ funds are safeguarded, the higher the bank’s
Capital Adequacy Ratio.
x Provisioning coverage ratio:
o Provision coverage ratio (PCR), on the other hand, refers to the percentage of funds created against
NPAs. A higher PCR ratio reflects that the bank has sufficient capital to withstand asset quality
pressures and will not need significant incremental capital in case of extreme stress. It has improved in
2022 as compared to last year. Hence, statement 2 is correct.
 14 www.visionias.in ©Vision IAS
.
 x Non-Performing Asset:
o NPA refers to a classification for loans or advances that are in default or are in arrears on scheduled
payments of principal or interest. The net non-Performing Assets ratio is the amount that is realized
 after the provision amount has been deducted from the gross non-performing assets. The net non-
Performing Assets ratio of banks has fallen during 2020-21 as compared to the previous year. Hence,
statement 1 is not correct.

Q 34.D
x Independent Departments
o Department of Atomic Energy (Parmanu Oorja Vibhag). Hence option 2 is correct.
o Department of Space (Antariksh Vibhag) Apex/Independent Office. Hence option 1 is correct.
x Apex/Independent Office
o Cabinet Secretariat (Mantrimandal Sachivalaya)
o President’s Secretariat (Rashtrapati Sachivalaya)
o Prime Minister’s Office (Pradhan Mantri Karyalaya)
o NITI Aayog (National Institution for Transforming India)
o National Security Council Secretariat (Rashtriya Suraksha Parishad Sachivalaya)
x Ministry of Law and Justice (Vidhi aur Nyaya Mantralaya)
o Department of Legal Affairs (Vidhi Karya Vibhag)
o Legislative Department (Vidhayee Vibhag). Hence option 4 is not correct.
o Department of Justice (Nyaya Vibhag)
x Ministry of Science and Technology (Vigyan aur Praudyogiki Mantralaya)
o Department of Science and Technology (Vigyan aur Praudyogiki Vibhag)
o Department of Scientific and Industrial Research (Vigyan aur Audyogik Anusandhan Vibhag)
o Department of Bio-Technology (Biotechnology Vibhag). Hence option 3 is not correct.

Q 35.D
x The 'Project Tiger' is a Centrally Sponsored Scheme (CSS) of the Ministry of Environment, Forests,
and Climate Change, and it has put the endangered tiger on an assured path of recovery by saving it from
extinction. Hence, statement 1 is correct.
x Project Tiger was launched in 1973 and was the first project of its kind in India to maintain the population
of tigers and protect them from poaching and other threats.
x The Core areas in the tiger reserves have the legal status of a national park or a sanctuary, whereas
the buffer or peripheral areas are a mix of forest and non-forest land, managed as a multiple-use
area. Hence, statement 2 is correct.
x India’s tiger population increased to 3,167 in 2022 from 2,967, recorded in 2018. This is a unique
distinction of having the maximum number of tigers in the world. Hence, statement 3 is correct.

Q 36.D
x The Scheme of Jan Shikshan Sansthan (JSS), formerly known as Shramik Vidyapeeth, has been
implemented through a network of NGOs in the country since March 1967. The first Shramik
Vidyapeeth was established in Mumbai [Worli] and was commissioned by the Bombay City Social Council
Education Committee, a voluntary organisation engaged in the field of adult education.
x After the success of the project, the Government of India developed a scheme for setting up a network of
such institutions in the country in a phased manner. Hence statement 3 is correct.
x With the transformation in the economic and social setup over the years, the role and scope of these
polyvalent educational institutions have widened manifold. In the changed scenario, the focus of Shramik
Vidyapeeth (SVP) was shifted from industrial workers in urban areas to the non-literates, neo-literates,
unskilled and unemployed youth, particularly from SC/ST/OBC/Minority/Divyang/Women throughout the
country, especially to underprivileged people in the rural areas. The SVPs were accordingly renamed as Jan
Shikshan Sansthan (JSS) w.e.f. the year 2000.
x The scheme of Jan Shikshan Sansthan was consequently transferred from the Ministry of Education
(erstwhile MHRD) to the Ministry of Skill Development and Entrepreneurship (MSDE) in July 2018.
The objectives of the Jan Shikshan Sansthan scheme are:
x To improve the occupational skills and technical knowledge of the non/neo literates and persons
having a rudimentary level of education upto 8th standard and other school dropouts beyond 8th
standard i.e. upto class 12th to raise their efficiency, increase their productive ability, and enhance their
livelihood opportunities. Hence statement 1 is correct.
 15 www.visionias.in ©Vision IAS
.
 x To identify and promote traditional skills in the districts through skilling/upskilling, thereby enabling
local trades to grow and creating new opportunities for the natives of the region. Hence statement 2
is correct.
 x To create a pool of master trainers working across the department/agencies of skill development through a
training/orientation programme.
x To collaborate and coordinate with other departments/agencies working in the field of skill development.
x To widen the range of knowledge and understanding of social, economic and political systems and create
awareness about the environment.
x To promote national values and to align with national programmes.
x To promote self-employment and facilitate getting financial support, including loans, for the target groups
through linkage with credit and consortium membership.
x Jan Shikshan Sansthans are registered under the Societies Registration Act, 1860. The affairs of Jan
Shikshan Sansthan are managed by the respective Board of Management approved by the Government of
India.

Q 37.C
x India has emerged as the second-largest producer of crude steel by replacing Japan. The biggest steel-
producing country is currently China.
x Top Producers of Steel:
o China (920 mt)
o India (109.3 mt)
o Japan (104.3 mt)
o USA (86.6 mt)
o South Korea (72.5 mt)
x Hence, option (c) is the correct answer.

Q 38.A
x Financial Intelligence Unit:
o It was established in India in 2004 as the central national agency responsible for receiving, processing,
analyzing, and disseminating information relating to suspect financial transactions.
o Financial Intelligence Unit-India (FIU-IND) is also responsible for coordinating and strengthening
efforts of national and international intelligence, investigation, and enforcement agencies in pursuing
global efforts against money laundering and related crimes.
o FIU-IND is an independent body reporting directly to the Economic Intelligence Council (EIC)
headed by the Finance Minister. Hence, statement 1 is correct and statement 2 is not correct.
x Financial Crimes Enforcement Network (FinCEN):
o It was set up in 1990.
o It serves as the leading global regulator in the battle against money laundering.
o It collects and analyzes information about financial transactions in order to combat domestic and
international money laundering, terrorist financing, and other financial crimes.

Q 39.D
x About Higher Education Financing Agency (HEFA):
x HEFA is a joint venture of the Ministry of HRD, GOI, and Canara Bank with agreed equity participation in
the ratio of 90.91% and 09.09% respectively. It was set up in 2017 to upgrade the education
infrastructure in higher education institutions under the Government of India. Hence statement 1 is
correct.
x HEFA is registered under the Companies Act 2013 as a Union Govt company and as Non–deposit
taking NBFC with RBI. Hence statement 2 is correct.
x VISION: To enable India’s premier educational institutions to excel and reach the top in global rankings
by financing building world-class infrastructure including R&D Infra.
x Functions:
x It will mobilize resources from the market by way of equity from individuals/corporates and by the issue of
bonds to finance the requirement. It provides financial assistance for the creation of educational
infrastructure and R&D in India’s premier educational Institutions. It encourages scientific and
technological developments by supporting R&D facilities for conducting high-quality research.
x How HEFA works? What are the advantages?
x The funding under HEFA will replace the current grant assistance by GOI for infrastructure projects
in premier educational institutions. Hence statement 3 is correct.
 16 www.visionias.in ©Vision IAS
.
 x All the Educational Institutions set up/funded and referred by concerned ministries would be eligible for
financing their capital expenditure from HEFA.
x HEFA would be able to fund a larger basket of institutions as compared to the grants approach. Top-class
 infrastructure can be created in a quick time so that the country realises the potential of its demographic
dividend in a faster time frame.
x “Revitalising Infrastructure and Systems in Education (RISE) by 2022”, is a major initiative
launched by GOI in the FY 2018-19 budget.
x HEFA’s scope under RISE has been greatly expanded from the initial objective of financing infrastructural
needs of select Higher Educational Institutions in India to the extent of Rs. 20,000 crores.

Q 40.B
x National Bank for Agriculture and Rural Development (hereinafter referred to as “NABARD”) is an
apex development financial institution established under the National Bank for Agriculture and Rural
Development Act, 1981.
o In order to assess the ground-level situation, NABARD undertakes frequent studies and surveys. One
such survey conducted by NABARD has been the ‘NABARD All India Rural Financial Inclusion
Survey (NAFIS) in 2016-17. NABARD now proposes to conduct the second round of survey,
“NABARD All India Rural Financial Inclusion Survey 2.0” (NAFIS 2.0).
o NAFIS 2.0 is planned to cover the livelihood aspects of the rural households in conjunction with
financial reach, access, products and transactions. Hence pair 1 is correctly matched.
x National Statistical Office (NSO) in its 77th round of survey, conducted during the period 1st January
2019 to 31st December 2019, carried out a survey on “Land and Livestock Holdings of Households and
Situation Assessment of Agricultural Households” in the rural areas of India with an integrated schedule of
enquiry. Prior to 77th round, Land and Livestock Holding Surveys (LHS) and Situation Assessment Survey
(SAS) of Agricultural Households used to be conducted as separate surveys in separate sets of
households. Hence pair 2 is correctly matched.
x Soil & Land Use Survey of India (SLUSI), earlier All India Soil & Land Use Survey (AISLUS)
established in 1958, is a subordinate office of the Department of Agriculture, Cooperation & Farmers
Welfare, Ministry of Agriculture & Farmers Welfare. The foundation of the national level soil surveys in
India was laid in 1953 when the Government of India invited Dr. F. F. Rickens, Soil Specialist from USA
to advice on organizing soil survey programmes in the country. Subsequently in 1955, the All India Soil
Survey Scheme was initiated under the Indian Agricultural Research Institute, New Delhi to carry out
reconnaissance soil surveys in the country.
o It is an apex organization in the country which deals with Soil Survey and Land Resource Mapping.
The SLUSI is a primarily engaged in conducting soil survey of different intensities in order to provide
scientific database for developmental programmes encompassing soil and water conservation planning,
watershed development, scientific land use planning etc. Hence pair 3 is not correctly matched.

Q 41.C
x The Ministry of Agriculture & Farmers’ Welfare is funding various projects for Crop Production
Forecasting, which includes FASAL Scheme (Forecasting Agricultural output using Space, Agro-
meteorology and Land based observations) and CHAMAN (Coordinated Horticulture Assessment
and Management using geo-informatics).
x FASAL is used for crop production forecasting of field crops while CHAMAN is for horticulture crops.
x In both the projects, Indian Space Research Organization (ISRO) has played a major role in developing
methodologies. However, currently the programmes are being operationalized by Mahalanobis
National Crop Forecast Centre (MNCFC) of Department of Agriculture & Farmers’ Welfare, with
technology support from ISRO.
x Nine crops which are assessed under FASAL are Rice, Wheat, Tur, Rabi Pulses, Rapeseed & Mustard, Rabi
Jowar, Cotton, Jute and Sugarcane.
x The seven crops which are being assessed under CHAMAN project are Potato, Onion, Tomato, Chilli,
Mango, Banana and Citrus. Hence option (c) is the correct answer.
x The KISAN project launched by Ministry of Agriculture & Farmers’ Welfare carried out various
pilot studies for evaluating the role of satellite technology in crop yield estimation.Based on these
studies and many consequent pilot studies carried out by the Ministry, currently, the satellite remote sensing
is being used for various operational applications under PMFBY, such as Smart Sampling for Crop Cutting
Experiments (CCEs), Yield Dispute Resolution and Direct Yield Estimation.

 17 www.visionias.in ©Vision IAS


.



.



.



.
 Q 51.C
x Recent Context: A plea which seeks to bring educational institutions like madrasas and vedic pathshalas
under the ambit of the Right to Education Act has been filed in the Delhi High Court.
 x The Constitution (Eighty-sixth Amendment) Act, 2002 inserted Article 21-A in the Constitution of
India to provide free and compulsory education of all children in the age group of six to fourteen years
as a Fundamental Right in such a manner as the State may, by law, determine.
x The Right of Children to Free and Compulsory Education (RTE) Act, 2009, which represents the
consequential legislation envisaged under Article 21-A, means that every child has a right to full time
elementary education of satisfactory and equitable quality in a formal school which satisfies certain essential
norms and standards.
x Institutions like madrasas and vedic pathshalas especially serving linguistic and religious minorities
are protected under Articles 29 and 30 and thereby are exempted under RTE Act 2009. Hence
statement 1 is correct.
x Schools are defined in under section 2(n) of the RTE Act, 2009: School” means any recognised
school imparting elementary education and includes—
o a school established, owned or controlled by the appropriate Government or a local authority;
o an aided school receiving aid or grants to meet whole or part of its expenses from the appropriate
Government or the local authority;
o a school belonging to specified category; and
o an unaided school not receiving any kind of aid or grants to meet its expenses from the appropriate
Government or the local authority;
x In 2019 amendment was done in RTE Act, 2009:
o There shall be a regular examination in the fifth class and in the eighth class at the end of every academic
year.
o If a child fails in the fifth or eighth class examination at the end of every academic year, he shall
be given additional instruction and granted opportunity for re-examination within a period of
two months from the date of declaration of the result.
o The appropriate Government may allow schools to hold back a child in the fifth class or in the
eighth class or in both classes, in such manner and subject to such conditions as may be prescribed, if
he fails in the re-examination referred to in sub-section (2). Hence statement 2 is correct.
o Provided that the appropriate Government may decide not to hold back a child in any class till the
completion of elementary education.
o No child shall be expelled from a school till the completion of elementary education.

Q 52.B
Transport and Marketing Assistance (TMA) for specified agriculture products scheme
x Recently, the Government enhanced the scope of the TMA scheme for specified agriculture products by
including dairy products in its purview and increasing the rates of assistance by 50% for exports by sea and
by 100% for exports by air.
x The scheme (Under the Ministry of Commerce) aims to provide assistance for the international
component of freight and marketing of agricultural produce. Hence statement 1 is not correct.
x This is done to mitigate the disadvantage of the higher cost of transportation of export of specified
agriculture products due to trans-shipment and to promote brand recognition for Indian agricultural products
in the specified overseas markets.
x Coverage-All exporters, duly registered with the relevant Export Promotion Council as per Foreign Trade
Policy, of eligible agriculture products shall be covered under this scheme. The assistance, at notified rates,
will be available for the export of eligible agriculture products to the permissible countries, as specified
from time to time.
x The pattern of Assistance: The assistance under TMA would be provided in cash through a direct
bank transfer as part of the reimbursement of freight paid. Hence statement 2 is correct.
x Condition for receiving assistance: The assistance shall be admissible only if payments for the exports are
received in Free Foreign Exchange through normal banking channels.

Q 53.C
x Graphite is a crystalline carbon that occurs in metamorphic and igneous rocks. In India, Arunachal
Pradesh accounts for 37 per cent of total resources, followed by Jammu and Kashmir (32 per cent) and
Odisha (10 per cent).

 21 www.visionias.in ©Vision IAS


.
 x Tungsten is a silvery white tough metal used as filaments in light bulbs and cathode ray tubes. Tungsten
resources are mainly distributed in Karnataka (42 per cent), Rajasthan (27 per cent) and Andhra Pradesh
(17 per cent).
 x Nickel occurs in nickeliferous limonite and in sulphide form. Around 92 per cent of Nickel resources
are found in Odisha, while the remaining 8 per cent are distributed in Jharkhand and Nagaland.
x Hence, option (c) is the correct answer.

Q 54.B
x Stockholm Convention is a global treaty to protect human health and the environment from Persistent
Organic Pollutants (POPs). t was opened for signature in 2001 in Stockholm (Sweden) and became
effective in 2004.
x Minamata Convention on Mercury was adopted at the Conference of Plenipotentiaries in 2013 to protect
human health and the environment from the adverse effects of mercury. Hence, statement 1 is not correct.
x In India, the National Implementation Plan for the convention has been developed through Global
Environment Facility funding. Hence, statement 2 is correct.
x Ministry of Environment, Forest and Climate Change serves as the focal point for the Stockholm
convention in India. Designated national authorities are in the Ministry of Agriculture and Farmers
Welfare and the Ministry of Chemicals and Fertilisers. Hence, statement 3 is correct.

Q 55.C
x Arjuna Award was instituted in 1961 by the Government of India to recognize outstanding achievement
in International sports events.
x It is given for good performance over a period of the previous four years and showing qualities
of leadership, sportsmanship and a sense of discipline. Hence, statement 2 is correct.
x It is the second-highest sporting honour of India, the highest being the Major Dhyan Chand Khel Ratna
Award. Hence, statement 1 is not correct.
x The Award consists of a statuette, a certificate, a ceremonial dress, and a cash prize. The Arjuna Award
cannot be given for a second time to the same person.
x It is awarded annually by the Ministry of Youth Affairs and Sports. Hence, statement 3 is correct.

Q 56.D
x The Ministry of Labour and Employment has launched the Platform for Effective Enforcement for
No Child Labour (PENCiL) portal to ensure effective implementation of the amended child labour
prohibition law.
x The portal integrates all the state governments with the central government for effective coordination and
convergence of various measures being taken for compliance with the Child Labour (Prohibition and
Prevention) Act. The enforcement of the Act would be supported by the portal, which enables the citizens
to raise a complaint and report incidents of child labour through effective tracking and monitoring
mechanisms.
x The platform also helps in monitoring the National Child Labour Project (NCLP), which has been aligned
with changes in the Act by the government. It is a rehabilitative scheme for child and adolescent labour.
The coverage of the scheme is being extended to all the districts of the country.
x Hence option (d) is the correct answer.

Q 57.A
x Indian Military Academy
o Founded in 1932, Indian Military Academy (IMA), Dehradun aims at the fullest development of
intellectual, moral and physical qualities of persons joining the Army as officers. Hence pair 1 is
correctly matched.
o The various modes of entry into IMA are: (a) on graduation from NDA; (b) on graduation from Army
Cadet College, which is a wing of the IMA itself; (c) direct entry graduate cadets, who qualify the Union
Public Service Commission Exam and get through the Service Selection Board; (d) for Technical
Graduate’s Course (TGC); and (e) under University Entry Scheme (UES) for engineering college
students in Final/ Pre-Final year of studies. The IMA also imparts training to gentlemen cadets from
friendly countries.
x Officers Training Academy: Established in 1963, the Officers Training School (OTS) was re-designated
as Officers Training Academy (OTA) from 1988 on the completion of 25 years of its existence.
o Its main task, before 1965, was to train gentlemen cadets for grant of Emergency Commission. From
1965 onwards, the Academy trains cadets for Short Service Commission. With the entry of women
 22 www.visionias.in ©Vision IAS
.
 officers in the Army since 1992, around 100 lady officers now get commissioned from OTA every year
in Army Service Corps, Judge Advocate General’s Department, Corps of Engineers, Signals and
Electrical and Mechanical Engineers.
 o Officers Training Academy, Gaya: The Academy was commissioned in 2011. The OTA, Gaya, also
imparts training to gentlemen cadets from friendly countries. Hence pair 2 is correctly matched.
x The College of Military Engineering (CME) at Pune is a premier technical institution conducting training
for personnel of the Corps of Engineers, other Arms, and Services, Navy, Air Force, Para Military Forces,
Police and Civilians, besides, personnel from friendly foreign countries. Hence pair 3 is not correctly
matched.
o CME is affiliated to Jawaharlal Nehru University (JNU) for the award of B.Tech. and M.Tech. degrees.
All India Council for Technical Education (AICTE) also recognises the graduate and postgraduate
courses run by the CME.

Q 58.B
x Zoological Survey of India (ZSI), which is under the Ministry of Environment, Forest and Climate
Change, is more than a century old committed to inventorying the faunal resources of the country right
from Protozoa to Mammalia.
x ZSI Headquarters at Kolkata and its 16 regional centres spread across the country are studying the Indian
fauna of all the states, UTs, and protected areas in different ecosystems.
x Altogether, 1,02,161 species have been documented by the ZSI from India, of which diversity of
Arthropods stands tall with 76,461 species. India contributes 6.52 percent of the global faunal diversity
with endemism of about 28 percent. Among the animals reported from the country, over 2,800 species are
protected under different schedules of the Wildlife (Protection) Act, 1972 for better conservation.
x About Arthropods:
o They make up about 75% of all animals on Earth and have a major role in maintaining ecosystems
as pollinators, recyclers of nutrients, scavengers, and food for other animals.
o Arthropods are invertebrates with joint legs.
o They include many animals we come across in our gardens, such as spiders, ants, centipedes and
slaters.
o Arthropods are divided into four major groups: insects; myriapods (including centipedes and
millipedes); arachnids (including spiders, mites and scorpions); crustaceans (including slaters,
prawn and crabs).
o Characteristic features of the phylum Arthropoda are:
9 Have bilateral symmetry.
9 Have a separate mouth and anus.
9 Have jointed body sections/appendages and have a hard exoskeleton (chitin).
9 Invertebrates.
x Hence option (b) is the correct answer.

Q 59.D
x The Asiatic Society is the oldest institution of learning in India and has made a seminal contribution in the
revival of Indian history and heralding its renaissance. It was founded by Sir William Jones, a revered
philologist, and scholar of Anglo-Welsh descent in 1784 at Calcutta.
x In 1984, the Government of India recognized the Asiatic Society as an institution of national importance
by an act of Parliament.
x Hence, option (d) is the correct answer.

Q 60.A
x What is the role of the Central Armed Police Forces (CAPFs)?
x The Central Armed Police Forces (CAPF) refers to seven security forces in India under the authority of the
Ministry of Home Affairs. Each of the seven has its own cadre of officers, but they are headed by officers
of the Indian Police Service.
x Under the Constitution, police and public order are state subjects. However, the Ministry of Home Affairs
(MHA) assists state governments by providing them support of the Central Armed Police Forces.
x The Ministry maintains seven CAPFs:
o the Central Reserve Police Force, which assists in internal security and counterinsurgency,
o the Central Industrial Security Force, which protects vital installations (like airports) and public sector
undertakings,
o the National Security Guards, which is a special counterterrorism force, and

 23 www.visionias.in ©Vision IAS


.
 o four border guarding forces, which are the Border Security Force, Indo-Tibetan Border Police, Sashastra
Seema Bal, and Assam Rifles. Hence option (a) is the correct answer.
x Organization and leadership
 o Central Armed Police Forces are organized with the primary role of border guarding for BSF, ITBP,
SSB; Security of sensitive establishments by CISF, Assisting Police to tackle Law & Order, Counter-
Terrorist Operations, Counter Naxal Operations by CRPF, NSG. Apart from the primary role, all CAPFs
are involved in assisting Police in Law & Order situations and also Army in Counter-Terrorist
Operations. BSF & CRPF have assisted the army during external aggression in the past. CAPFs work
along with both Army & Police in different roles assigned to them.
o Central Armed Police Forces personnel also serve in various important organizations such as Research
and Analysis Wing (RAW), Special Protection Group (SPG), National Investigation Agency (NIA),
Intelligence Bureau (IB), Central Bureau of Investigation (CBI), National Disaster Response Force
(NDRF), Narcotics Control Bureau (NCB) State Armed Police Force (Jharkhand Jaguars, Bihar
Military Police, UP/MP STF, DRG, IRB, Chhattisgarh Armed Police etc.) on deputation and have
attachment/training in various levels/formations/courses along with the Indian Army. Their role and
performance, therefore, assumes a great significance due to the special features of an emergency force
which is pressed in aid to the civil power to perform multiple roles in extremely difficult situations.

Q 61.D
x About India’s Drug Regulation Ecosystem The Indian Drug Regulatory System originated from the
Drugs & Cosmetics Act, of 1940. It regulates the import, manufacture, distribution and sale of drugs and
cosmetics. The Central Drugs Standard Control Organization (CDSCO) is the Central Drug
Authority for discharging functions assigned to the Central Government under the Drugs and
Cosmetics Act. Hence statement 1 is correct.
x The 1940 act and its corresponding rules allow the Centre and State to regulate different aspects of the drug
ecosystem. Under the Drug and Cosmetics Act, the regulation of the manufacture, sale, and
distribution of Drugs is primarily the concern of the State authorities. Hence statement 2 is correct.
x What is the process of Drug Regulation?
x CDSCO has been entrusted with the responsibility for the approval of new drugs, and the conduct of
clinical trials in the country, as well as laying down the standards for drugs. Hence statement 3 is
correct.
x It also controls the quality of imported drugs, has oversight over the SDRAs, and an advisory role in
ensuring uniformity in the enforcement of the Drug Control Act.
x Applications for approval of New Drugs are evaluated by the 12 Subject Expert Committee (SEC) (formerly
referred to as New Drug Advisory Committees (NDAC)), consisting of experts usually drawn from
Government Medical Colleges and Institutes across India.

 24 www.visionias.in ©Vision IAS


.
 Q 62.D
x The Indian Coast Guard (ICG) is an armed force that protects India's maritime interests and enforces
maritime law. Indian Coast Guard carries out overall responsibility for coastal security in territorial waters
 including contiguous zone and exclusive economic zone.
x The Coast Guard works in close cooperation with the Indian Navy, the Department of Fisheries, the
Department of Revenue (Customs) and the Central and State police forces. It operates under the Union
Ministry of Defence. Hence statement 2 is not correct.
x The coastal police's jurisdiction extends up to 12 nautical miles territorial waters except in Port Blair
where jurisdiction of 10 designated coastal police stations has been extended up to India's Exclusive
Economic Zone (EEZ) or 200 nautical miles. Hence statement 1 is not correct.
x In the aftermath of the 2008 Mumbai terror attacks, the coastal police is primarily responsible for prevention
of smuggling, collection of intelligence, handling law and order situations, disaster management as well as
providing additional line of defence along the coastline.
x United Nations Convention on the Law of the Sea (UNCLOS), provide an international framework for
analysis of private maritime security. UNCLOS sections the oceans, splitting marine areas into five main
zones, each with a different legal status: Internal Waters, Territorial Sea, Contiguous Zone, Exclusive
Economic Zone (EEZ) and the High Seas.
o Internal Waters: Internal Waters include littoral areas such as ports, rivers, inlets and other marine
spaces landward of the baseline (low-water line) where the port state has jurisdiction to enforce
domestic regulations. Enforcement measures can be taken for violations of static standards while in port
as well as for violations that occurred within the coastal state’s maritime zones and beyond. However,
foreign vessels are not usually held to non-maritime or security port state laws so long as the activities
conducted are not detrimental to the peace and security of the locale.
o Territorial Sea: In the Territorial Sea, a coastal state has unlimited jurisdiction over all (including
foreign) activities unless restrictions are imposed by law. All coastal states have the right to a territorial
sea extending 12 nautical miles from the baseline.
o Contiguous Zone: The Contiguous Zone is an intermediary zone between the territorial sea and the
high seas extending enforcement jurisdiction of the coastal state to a maximum of 24 nautical miles
from baselines for the purposes of preventing or punishing violations of customs, fiscal, immigration
or sanitary (and thus residual national security) legislation.
o Exclusive Economic Zone (EEZ): The EEZ is another intermediary zone, lying between the territorial
sea (12 nautical miles) and the high seas to the maximum extent of 200 nautical miles. Although high
seas freedoms concerning general navigation principles remain in place, in this zone the coastal state
retains exclusive sovereignty over exploring, exploiting and conserving all natural resources. The
coastal state therefore can take action to prevent infringement by third parties of its economic assets in
this area including, inter alia, fishing, bio-prospecting and wind-farming. In order to safeguard these
rights, the coastal state may take necessary measures including boarding, inspection, arrest and judicial
proceedings, as may be necessary to ensure compliance with the international laws and regulations.
o High Seas: The High Seas, which lie beyond 200 nautical miles from shore, are to be open and freely
available to everyone, governed by the principle of equal rights for all. In agreeing to UNCLOS, all
state parties acknowledged that the oceans are for peaceful purposes as the Convention’s aim was to
maintain peace, justice and progress for all people of the world. On the High Seas, no state can act or
interfere with justified and equal interests of other states.

Q 63.D
x Natural Farming is a chemical-free farming system rooted in Indian tradition enriched with a modern
understanding of ecology, resource recycling and on-farm resource optimization. It is considered an
agroecology-based diversified farming system which integrates crops, trees, and livestock with functional
biodiversity. It is largely based on on-farm biomass recycling with major stress on biomass mulching, use
of on-farm cow dung-urine formulations; maintaining soil aeration and exclusion of all synthetic chemical
inputs. Natural farming is expected to reduce dependency on purchased inputs. It is considered a cost-
effective farming practice with scope for increasing employment and rural development.
x There are many working models of natural farming all over the world, the zero budget natural farming
(ZBNF) is the most popular model in India. Natural Farming improves soil fertility, and environmental
health as well as helps in the reduction of greenhouse gas emissions, and also promises the
enhancement of farmers’ income. In broad terms, Natural Farming can be considered as a prominent
strategy to save the planet Earth for future generations. It has the potential to manage the various farmland
practices and hence sequester the atmospheric carbon in the soils and plants, to make it available for
plants. Hence statement 2 is correct.
 25 www.visionias.in ©Vision IAS
.
 x Bhartiya Prakratik Krishi Padhati (BPKP) is a zero external input system of organic agriculture
largely relying on the principles of Zero Budget Natural Farming (ZBNF) is aimed at promoting
traditional indigenous practices which gives freedom to farmers from externally purchased inputs and is
 largely based on on-farm biomass recycling with major stress on biomass mulching, use of on-farm cow
dung-urine formulations (such as Bijamrit and Jivamrit); time to time working for soil aeration and
exclusion of all synthetic chemical inputs directly or indirectly.
o To promote natural farming across the country, the Centre has launched the National Mission on Natural
Farming (NMNF) by upscaling the Bharatiya Prakritik Krishi Paddhati (BPKP) scheme.
o NMNF will cover a 7.5 lakh hectare area by developing 15,000 clusters in the next four years with a
total budget outlay of ₹1,584 crore (Centre’s share).
o With NMNF, the government proposes to touch one crore farmers along the Ganga belt and in other
rainfed parts of the country. Hence statement 1 is correct.
o Under NMNF, farmers will receive financial assistance of ₹15,000 per hectare per year for three years
for the creation of on-farm input production infrastructure.

x Hence statement 3 is correct.

Q 64.C
x The Ministry of Women and Child Development along with the Bill & Melinda Gates Foundation
(BMGF) launched the Bharatiya Poshan Krishi Kosh (BPKK) to promote and reinforce healthy
dietary practices at the basic individual and community level and sustainably tackle malnutrition.
"Bhartiya Poshan Krishi Kosh" project has two components – Development of a Food Atlas and
Documentation of promising practices for Jan-Andolan for POSHAN Abhiyaan. Hence statement 2 is
correct.
x The Agro-Food Atlas acts as a repository of diverse crops across 128 agro-climatic zones of the
country and has three parts- crops currently being grown, agro-ecological conditions (soil, organic carbon
content, ground water availability, etc.) and guidance on how a greater diversity of crops could be
encouraged in a particular district or block to promote dietary diversity and nutrition. Hence statement 1
is correct.
x The project includes diverse data sources like the National Sample Survey, Agri-Census, Soil Health Cards,
ISRO’s Advanced Wide Field Sensor (AWiFS) and NASA’s Moderate Resolution Imaging Spectro-Radio
Metre.
x The project also documents social, behavioral and cultural practices that promote and reinforce healthy
dietary behaviours. Identification of promising practices with the help of a multi-disciplinary group of
experts and the development of a tool kit to disseminate the best strategies for Social and Behavioural
Change Communication, specific to population groups in those regions, is also a part of it.

Q 65.C
x Vishnuprayag is one of the Panch Prayag (five confluences) of the Alaknanda River, and lies at the
confluence of Alaknanda River and Dhauliganga River, in Chamoli district in the Indian state of
Uttarakhand.
x Nandaprayag is a town and a nagar panchayat in Chamoli district in the Indian state of Uttarakhand.
Nandaprayag is one of the Panch Prayag of Alaknanda River and lies at the confluence of the Alaknanda
River and Nandakini River.

 26 www.visionias.in ©Vision IAS


.
 x Karnaprayag is a city and municipal board in the Chamoli District in the Indian state of Uttarakhand.
Karnaprayag is one of the Panch Prayag of Alaknanda River, situated at the confluence of the Alaknanda
and Pindar River.
 x Rudraprayag is a town and a municipality in Rudraprayag district in the Indian state of Uttarakhand.
Rudraprayag is one of the Panch Prayag of Alaknanda River, the point of confluence of rivers Alaknanda
and Mandakini. Kedarnath, a Hindu holy town is located 86 km from Rudraprayag.
x Devprayag is a town and a nagar panchayat in Tehri Garhwal district in the state of Uttarakhand, India, and
is one of the Panch Prayag of Alaknanda River where Alaknanda, Saraswati River and Bhagirathi rivers
meet and take the name Ganga.
x Hence option (c) is the correct answer.

Q 66.B
x CHAMPION SERVICES SECTOR SCHEME (CSSS)-A Central Sector scheme launched for the
period 2019- 20 to 2023-24.
x Objective - To enhance the competitiveness of India’s service sectors thereby creating more jobs in India,
contributing to a higher GDP and export of services to global markets.
x Implementation - Three-tier monitoring mechanism with the concerned Ministry/ Department, the
Department of Commerce (The Ministry of Commerce and Industry) and the Committee of Secretaries
chaired by the Cabinet Secretary.
x 12 Champion services sectors - Focused attention on the promotion of Environmental, Legal, Audio -
Visual, Communication, Education, Financial, Construction and Related Engineering
Services, Accounting and Finance Services, Transport & Logistics, IT & ITeS, Medical Value Travel,
Tourism & Hospitality Services. Hence option (b) is the correct answer.
x Benefits - Promote innovation, boost services exports, skill training and address sectoral and cross-cutting
issues including regulatory reforms, service standards, data protection, etc.

Q 67.B
x The Indian Tsunami Early Warning Centre (ITEWC) established at the Indian National Centre for Ocean
Information Sciences, (INCOIS), Hyderabad, under the Ministry of Earth Sciences is the national
authority to issue tsunami advisories for India. Hence, statement 1 is not correct.
x It is equipped to issue warnings within ten minutes of the occurrence of earthquakes. Hence, statement
2 is correct.
x The ITEWC functions as an approved Tsunami Service Provider of the Indian Ocean Tsunami Warning &
Mitigation System (IOTWMS) that is an integral part of the Global Tsunami Warning and Mitigation
System, established and coordinated by the Intergovernmental Oceanographic Commission (IOC) of
UNESCO.

 27 www.visionias.in ©Vision IAS


.
 Q 68.A
x The Ministry of Agriculture and Farmers Welfare launched the Sub Mission on Agricultural Mechanization
(SMAM) in 2014-15.
 x Under it, subsidy is provided for purchase of various types of agricultural equipment and machinery to
the extent of 40-50% for States other than NER (North Eastern Region) States and for NER States it is
100% limited to Rs.1.25 lakhs per beneficiary. Hence statement 1 is correct.
x The agriculture ministry has also developed a Multilingual Mobile App, 'CHC (Custom Hiring Centres)-
Farm Machinery' which connects farmers with Custom Hiring Service Centres situated in their locality.
x Aim:
o Increasing the reach of farm mechanization to small and marginal farmers and to the regions and
difficult areas where farm power availability is low.
x Objectives:
o Promoting ‘Custom Hiring Centres’ and ‘Hi-tech Hubs of High-Value Machines’ to offset the adverse
economies of scale arising due to small and fragmented landholding and high cost of individual
ownership.
o Creating awareness among stakeholders through demonstration and capacity building activities.
o Ensuring performance testing and certification of agricultural machines at designated testing centres
located all over the country.
x Role of Panchayati Raj Institutions (PRIs):
o The State Government and other designated implementing agencies, to the extent possible, will ensure
active participation of the Panchayati Raj Institutions (PRIs) in the implementation of this Mission.
o PRIs may also be involved in publicising the demonstrations and training of farm equipment and in
ensuring participation of farmers from nearby areas for widespread dissemination of technology.
x Identification of Beneficiaries: Individual farmer, SHG/UG of farmers/ Cooperative Societies of
Farmers/FPOs/Entrepreneurs will be identified by the district nodal agencies. Hence statement 2 is not
correct.
x Recently, the Prime Minister has flagged off 100 Kisan drones under SMAM in different cities and
towns of India to spray pesticides in farms across India.
o The Kisan drone will have an unmanned tank filled with insecticides and nutrients.
o The drones are expected to have a high capacity of 5 to 10kg.
o The drone will spray the same amount of pesticide on about one acre of land in just 15 minutes.
o It will save time, will require less effort and spraying will be done uniformly.
o They will also be used to carry vegetables, fruits, fish, etc to the markets from the farms.
o These items will be supplied directly to the market with minimal damage, consuming lesser time,
resulting in more profits to farmers and fishermen.
o For promotion of Kisan Drones, provisions have been made for financial assistance under the guidelines
of Sub-Mission on Agricultural Mechanization (SMAM).
Q 69.C
x Mission Sagar is India’s helping hand across the Indian Ocean Region. Responding to their requests for
assistance in dealing with the Covid-19 pandemic, Government of India has sent Indian Naval Ship to
Maldives, Mauritius, Madagascar, Comoros and Seychelles, carrying onboard Medical Assistance Teams,
consignments of COVID related essential medicines and essential food items. Mission Sagar’ is inspired by
the Prime Minister’s vision of SAGAR - Security and Growth for All in the Region. Hence, pair 1 is not
correctly matched.
x Indian Navy has launched Operation “Samudra Setu” - meaning “Sea Bridge”, as a part of national
effort to repatriate Indian citizens from overseas. Indian Naval Ships Jalashwa and Magar are presently
en-route to the port of Malè, the Republic of Maldives to commence evacuation operations. The evacuated
personnel would be provided with the basic amenities and medical facilities during the sea-passage. Hence,
pair 2 is not correctly matched.
x Operation Warp Speed is a public-private partnership, initiated by the Federal Government of the United
States, to facilitate and accelerate the development, manufacturing, and distribution of COVID-19 vaccines,
therapeutics, and diagnostics. Hence, pair 3 is correctly matched.

Q 70.B
x Vivad Se Vishwas scheme
o It was launched in 2020 after the approval of the Parliament with the objective of reducing litigation.
o Direct Tax Vivad se Vishwas Act 2020 was enacted in 2020 under which the declarations for settling
disputes are being filed. Depending on the type of the pending dispute, a proportion of the total tax,
interest, and penalty demanded, needs to be paid under the schemes for settlement.
 28 www.visionias.in ©Vision IAS
.
 o The scheme aims to benefit those whose tax demands are locked in dispute in multiple forums.
o It provides for the settlement of disputed tax, disputed interest, disputed penalty, or disputed fees
in relation to an assessment or reassessment order.
 o The Vivad Se Vishwas scheme offers to settle pending direct tax-related disputes.
o The dispute is settled on payment of 100% of the disputed tax and 25% of the disputed penalty
or interest or fee.
o The taxpayer is granted immunity from levy of interest, penalty, and institution of any proceeding for
prosecution for any offense under the Income-tax Act in respect of matters covered in the declaration.
o Disputes related to wealth, securities transactions, commodities transaction tax, and the
equalization levy are not covered under the scheme. Hence, option (b) is the correct answer.

Q 71.B
x The National Authority for Chemical Weapons Convention (NACWC) was set up as an office of the
Cabinet Secretariat, Government of India to fulfill, on behalf of the Government of India, the obligations
under the Chemical Weapons Convention (CWC) and to act as the national focal point for effective
liaison with the Organisation for the Prohibition of Chemical Weapons (OPCW) and other state parties
on matters relating to the Convention. Hence, statement 1 is not correct and statement 2 is correct.
x In 2000, the Chemical Weapons Convention Act was passed by the Parliament which came into force in
2005.
x India is one of the four non-elected Members of the Executive Committee (consisting of 41 members from
all over the world) of the OPCW.

Q 72.D
x Recent context: The government has surpassed the formal employment creation target set under the
Atmanirbhar Bharat Rozgar Yojana (ABRY), which incentivises employers for creating jobs.
x Aatmanirbhar Bharat Rozgar Yojana (ABRY) was announced as a part of the Aatmanirbhar Bharat 3.0
package to boost the economy, increase the employment generation in the post-COVID recovery
phase, and to incentivize the creation of new employment along with social security benefits and the
restoration of lost employment during the COVID-19 pandemic.
x ABRY, being implemented through the Employees Provident Fund Organisation (EPFO), reduces the
financial burden of the employers of various sectors/industries and encourages them to hire more workers.
x Under ABRY, the Government of India is crediting for a period of two years both the employees’ share
(12% of wages) and employers share’ (12% of wages) of contribution payable or only the employees’ share,
depending on the employment strength of the EPFO-registered establishments.
x Under ABRY, benefits are provided to every establishment registered with EPFO and their new employees
(earning wage less than Rs. 15,000/- per month) if the establishments take new employees on or after
1.10.2020 and upto 30th June, 2021 or those who lost jobs between 01.03.2020 to 30.09.2020.
x The scope of the scheme i.e. last date for registration of new employees under the scheme, has been extended
from 30th June 2021 to 31st march 2022. Approximately 71.8 lakh employees are likely to get benefits
during the scheme period. The beneficiaries registered upto 31st March, 2022 will continue to receive the
benefits for 2 years from the date of registration under the scheme.
x It has helped formalise informal employment and create new employment opportunities during and post-
COVID 19 pandemic.
x Hence option (d) is the correct answer.

Q 73.B
x National Smart Grid Mission (NSGM) has been established by Govt. of India to accelerate Smart Grid
deployment in India. NSGM has been in operational since January 2016 with dedicated team.
o The primary aim of the Smart Grids is to improve reliability of the Electricity networks and make
the grid amenable to renewable energy inputs through distributed generation. Hence statement 2 is
not correct.
o Increased efficiencies with Smart Grid and Smart Meters empower the consumers to manage their
electricity consumption in a better manner and help them in reducing their bills.
o The NSGM also envisages capacity building initiatives for Distribution Sector personnel in the field of
Smart grids.
x NSGM has its own resources, authority, functional & financial autonomy to plan and monitor
implementation of the policies and programs related to Smart Grids in the country. Hence statement 3 is
correct.

 29 www.visionias.in ©Vision IAS


.
 x NSGM is housed under Ministry of Power (MoP) considering the fact that most of the prominent
stakeholders (DISCOMs, Regulators, Electrical Manufactures, CEA etc.) for Smart Grid are associated with
MoP. Other concerned Ministries like MNRE and MoHI would also be associated with the mission. Hence
 statement 1 is not correct.
x The Smart Grid Knowledge Centre being developed by POWERGRID with funding from MoP will act as
a Resource Centre for providing technical support to the Mission in all technical matters, including
development of technical manpower, capacity building, outreach, suggesting curriculum changes in
technical education etc.

Q 74.D
x The Integrated Power Development Scheme (IPDS) in 2014 by Ministry of Power with the objectives of:
o Strengthening of sub-transmission and distribution network in the urban areas. Hence statement 1 is
not correct.
o Metering of distribution transformers/feeders/consumers in the urban areas. IT enablement of
distribution sector and strengthening of distribution network under Restructured Accelerated Power
Development and Reforms Programme (R-APDRP).
x Significance of the scheme:
o The scheme will help in reduction in AT&C losses; establishment of IT enabled energy accounting/
auditing system, improvement in billed energy based on metered consumption and improvement in
collection efficiency.
x Power Finance Corporation Limited (PFC) has been designated as the Nodal Agency for operationalization
and implementation of the scheme under the overall guidance of Ministry of Power. Hence statement 3 is
correct.
x Eligible Utilities: All Discoms will be eligible for financial assistance under the scheme. Hence statement
2 is correct.

Q 75.C
x The Union Cabinet has approved the Production Linked Incentive (PLI) Scheme to promote domestic
manufacturing of critical Key Starting Materials/Drug Intermediates and Active Pharmaceutical Ingredients
in the country. The scheme is applicable only for greenfield projects. Hence statement 1 is correct.
x Aim: The PLI scheme aims to promote domestic manufacturing of critical Key Starting Materials
(KSMs)/Drug Intermediates and Active Pharmaceutical Ingredients (APIs) in the country.Funding: Under
the scheme financial incentive will be given to eligible manufacturers of identified 53 critical bulk drugs on
their incremental sales over the base year (2019-20) for a period of 6 years. Financial incentives will be
based on incremental sales over the base year 2019-20 for 6 years to selected manufacturers.
x Impact: PLI scheme will reduce India's import dependence on other countries for critical KSMs/Drug
Intermediates and APIs.
x Implementation: The scheme will be implemented through a Project Management Agency (PMA) to
be nominated by the Department of Pharmaceuticals. Hence statement 2 is correct.

Q 76.B
x Hon'ble Prime Minister Shri Narendra Modi launched Saansad Adarsh Gram Yojana (SAGY) on
11th October 2014 on the birth anniversary of Lok Nayak Jai Prakash Narayan at Vigyan Bhawan,
New Delhi.
x The goal was to develop three Adarsh Grams by March 2019, of which one was to be achieved by 2016.
Thereafter, five such Adarsh Grams (one per year) will be selected and developed by 2024.
x Inspired by the principles and values of Mahatma Gandhi, the Scheme places equal stress on nurturing
values of national pride, patriotism, community spirit, and self-confidence and on developing
infrastructure.
x Gram Panchayat would be the basic unit for development. It will have a population of 3000-5000 in
plain areas and 1000-3000 in hilly, tribal and difficult areas.
 30 www.visionias.in ©Vision IAS
.
 x Member of Parliament (MP) will identify one Gram Panchayat to be taken up immediately, and two
others to be taken up a little late. Hence statement 1 is not correct.
x The scheme will be implemented through a village development plan that would be prepared for every
 identified gram Panchayat.
x The goal of Saansad Adarsh Gram Yojana (SAGY) is to translate this comprehensive and organic vision of
Mahatma Gandhi into reality, keeping in view the present context.
x The main objectives of SAGY are:
o To trigger processes which lead to the holistic development of the identified Gram Panchayats
o To substantially improve the standard of living and quality of life of all sections of the population
through improved basic amenities
o Higher productivity
o Enhanced human development
o Better livelihood opportunities
o Reduced disparities
o Access to rights and entitlements
o Wider social mobilization
o Enriched social capital
x The scheme is implemented by the Ministry of Rural Development. Hence statement 2 is correct.

Q 77.C
x With a view to enabling an area based development approach, a new scheme Pradhan Mantri Adarsh Gram
Yojana (PMAGY) was launched during 2009-10.
x To ensure integrated development of all villages having a total population ≥500 and with more than 50%
persons belonging to the Scheduled Castes into “model villages” (by 2024- 25). Hence, statement 1 is
correct.
x The principal objective of the Scheme is the integrated development of SC-majority Villages:
o Primarily through the convergent implementation of the relevant Central and State/UT Government
Schemes; and
o By taking up identified activities, which do not get covered under the existing Central and State
Government Schemes, through ‘Gap-filling’ funds provided as Central Assistance to the extent of
Rs.20.00 lakh per village. Hence, statement 2 is correct.
x It is implemented by the Ministry of Social Justice and Empowerment. Hence, statement 3 is not correct.

Q 78.A
x Study titled “Foundational Learning Study 2022” was jointly conducted by the Union Ministry of
Education and the National Council of Educational Research and Training (NCERT). Hence
statement 1 is correct.
x It is a one-of-a-kind study to arrive at the benchmarks in Foundational Literacy and Numeracy (FLN)
under the FLS 2022.
x FLN is the ability to read with meaning and perform basic math calculations by the end of Class 3.
Hence statement 2 is not correct.
x The National Education Policy (NEP) 2020 accords the highest priority to achieving FLN among all
children by 2025.
x It is also the largest study that assessed the learning levels of approx. 86,000 grade 3 students from
10,000 schools across India. It is the only study that has been conducted in 20 different languages.
x Parameters for foundational literacy skills included oral language comprehension, reading comprehension,
oral reading fluency with comprehension, etc.
x Parameters for foundational numeracy skills included number identification & comparison, number
operations, data handling etc.

 31 www.visionias.in ©Vision IAS


.
 Q 79.C
x SAKSHAM – Scholarship Scheme for Differently Abled Children –aims at providing encouragement
and support to differently-abled children to pursue technical education. Hence statement 1 is correct.
 x Scholarships amounting to ₹ 5 crores per annum as tuition fees and incidentals are to be provided to
needy and meritorious students for pursuing technical education at AICTE-approved institutions.
Hence statement 2 is correct.
x This is to help them achieve their college goals, despite learning issues, environmental challenges, or
medical bills.

Q 80.C
x The Central Board of Secondary Education was established in 1929.
o It is a Board of Education for public and private schools, under the Union Government.
x The Board shall have the following powers namely:
o To conduct examinations;
o To admit candidates to its examinations;
o To recognize institutions for the purpose of its examinations;
o To inspect or call for inspection report on the condition of institutions recognized or applying for
recognition;
o To prescribe courses of instruction for recognized institutions;
o To grant diplomas or certificate of having passed the examinations of the Board to persons who – have
perused a course of study in any institution recognized by that Board, or to teachers, or have studied
privately under conditions laid down in the regulations.
o To demand and receive such fees as may be prescribed in the regulations.
o To supervise the residence, health, and discipline of the students of the recognised institution and to
make arrangements for promoting their general welfare.
o To do all such other acts and things as may be requisite to further the objects of the Board as a body
constituted for regulating and supervising Secondary Education.
x National Bal Bhavan (NBB), established in 1956, is an autonomous institution funded by the Ministry
of Education.
o Bal Bhavan as a movement is spread across the length and breadth of the country with 134 affiliated
Bal Bhavans and Bal Kendras. In addition there are 48 Bal Bhavan Kendras and a Bal Bhavan at Mandi
village in Delhi.
o National Bal Bhavan aims at enhancing the creative potential of children by providing them various
activities, opportunities and common platform to interact, experiment, create and perform according to
their age, aptitude and ability.
x The National Council of Educational Research and Training (NCERT) provides academic and
technical support for the qualitative improvement of school education. The NCERT was established in
1961 as an apex national body to lead qualitative changes in school education. NCERT has been playing
an advisory role in guiding central and state governments in formulating policies, acts, and government
programs. It has played a crucial role in the development of national policies on education (1968- 1986)
and national curriculum frameworks.
o NCERT has been designing and offering innovative and need-based courses for teachers, teacher
educators, and counselors.
o It is a unique institution in the country, conducting research, preparing skilled educational professionals,
and developing curriculum and curricular materials.
o The major constituent units of the NCERT are the National Institute of Education (NIE), New Delhi;
Central Institute of Education Technology (CIET), New Delhi; Pandit Sunder Lal Sharma Central
Institute of Vocational Education (PSSCIVE), Bhopal; and five regional institutes of education (RIEs)
at Ajmer, Bhopal, Bhubaneswar, Mysuru and Umiam (Meghalaya).
x National Institute of Open Schooling (NIOS) is an ‘Open School’ to cater to the needs of a heterogeneous
group of learners up to pre-degree level. It was started as a project with in-built flexibilities by the Central
Board of Secondary Education (CBSE) in 1979.
o The Ministry Human Resource Development (now Ministry of Education) set up the National
Open School (NOS) in 1989.
o In July 2002, the Ministry amended the nomenclature of the organisation from the National Open
School to the National Institute of Open Schooling (NIOS) with a mission to provide relevant
continuing education at school stage, up to pre-degree level through open learning system to prioritised
client groups as an alternative to formal system, in pursuance of the normative national policy
documents and in response to the need assessments of the people, and through it to make its share of
 32 www.visionias.in ©Vision IAS
.
 contribution; to universalisation of education; to greater equity and justice in society; and to the
evolution of a learning society.
x Hence option (c) is the correct answer.

Q 81.B
x Rashtriya Vayoshri Yojana (RVY) is a scheme for providing Physical Aids and Assisted-living
Devices for only Senior citizens belonging to the BPL category. The main criteria for the senior citizens
to get full benefits of the Rashtriya Vayoshri Yojana (RVY) scheme is that they must belong to a BPL
family and should hold a valid BPL card issued by the concerned authority. Hence, statement 1 is correct.
x This is a Central Sector Scheme, fully funded by the Central Government. Hence, statement 2 is correct.
x The expenditure for implementation of the scheme will be met from the "Senior Citizens' Welfare Fund".
x The Scheme will be implemented through the sole implementing agency - Artificial Limbs Manufacturing
Corporation (ALIMCO), a PSU under the Ministry of Social Justice and Empowerment. Hence,
statement 3 is not correct.
x Under the scheme, physical aid will be provided only to the senior citizens of the nation.
x This implies those who are aged above 60 years will get free assisted living aids and physical devices which
are required for their sustainability. Also the govt. has selected the list of cities where the scheme will be
implemented.
x Eligibility
o Senior Citizens, belonging to the BPL category and suffering from any of the age-related
disability/infirmity viz. Low vision, Hearing impairment, Loss of teeth, and Locomotor disability will
be provided with such assisted-living devices which can restore near normalcy in their bodily functions,
overcoming the disability/infirmity manifested. The Scheme is expected to benefit all Senior Citizens
across the country.
x Salient features of the scheme
o Free-of-cost distribution of the devices, commensurate with the extent of disability/infirmity that is
manifested among the eligible senior citizens.
o In case of multiple disabilities/infirmities manifested in the same person, assistive devices will be given
in respect of each disability/impairment.
o Artificial Limbs Manufacturing Corporation (ALIMCO) will undertake one-year free
maintenance of the aids & assisted living devices.
o Beneficiaries in each district will be identified by the State Governments/UT Administrations
through a Committee chaired by the Deputy Commissioner/District Collector.
o As far as possible, 30% of the beneficiaries in each district shall be women.
o The State Government/UT Administration/District Level Committee can also utilize the data of BPL
beneficiaries receiving Old Age Pension under the NSAP or any other Scheme of the State/UT for
identification of senior citizens belonging to BPL category.
o The devices will be distributed in camp mode.

Q 82.D
x The Government of India has decided to launch a new sub-scheme named “Aajeevika Grameen Express
Yojana (AGEY)” as part of the Deendayal Antyodaya Yojana – National Rural Livelihoods Mission (DAY-
NRLM).
x The Self-Help Groups under DAY-NRLM will operate road transport services in backward areas.
This will help to provide safe, affordable and community-monitored rural transport services to
connect remote villages with key services and amenities (such as access to markets, education and
health) for the overall economic development of backward rural areas.
o The State Rural Livelihood Missions (SRLMs) will do a feasibility study and traffic survey in the
selected blocks to identity the routes and the number and capacity of vehicles which can be operated on
a sustainable basis.
o The SRLMs will be coordinating with State Transport Department for the issue of a permit for the
vehicle. The SHG member operating the vehicle shall ensure that all necessary legal and statutory
requirements such as valid permits, road tax permits, valid insurance policies etc. are met.
o All vehicles under the scheme shall have a defined colour code and carry AGEY branding to ensure
their identity and avoid diversion to other routes.
x Hence, option (d) is the correct answer.

 33 www.visionias.in ©Vision IAS


.
 Q 83.C
x Economic Census (EC):
o It is the complete count of all establishments/units located within the geographical boundaries of
 the country.
o It is conducted every five years and is very crucial for framing of policies and planning for the
government and other organizations. Hence option (c) is the correct answer.
o It was held in 1978 for the first time and this is the 7th Census which will provide disaggregated
information on various operational and structural aspects of all establishments in the country.
o Agriculture is not covered in the Economic census (Only the Growing of agricultural crops and
plantations should not be counted as establishments for the purpose of this Economic Census).
o What is Establishment?
9 An establishment is a unit situated in a single location in which predominantly one kind of economic
activity is carried out such that at least a part of the goods and/or services produced by the unit goes
for sale (i.e. the entire product is not for sole consumption).
o Which are the Bodies involved in carrying it out?
9 The 7th Economic Census -2019 is being conducted by the Ministry of Statistics and Program
Implementation (MoSPI).
9 MoSPI has partnered with Common Service Centres, CSC e-Governance Services India Limited,
and a Special Purpose Vehicle (SPV) under the Ministry of Electronics and Information Technology
(MeitY) as the implementing agency for the 7th EC.
o Parent Act: The data is collected through door-to-door surveys of each household and commercial
establishment under the provisions of the Collection of Statistics Act 2008.

Q 84.C
x The Central Electricity Authority (CEA) is a statutory organization constituted under Section 3 (1) of the
repealed Electricity (Supply) Act, 1948 and continued under Section 70 of the Electricity Act, 2003. Hence
statement 1 is correct.
x It was established as a part time body in 1951 and made a full time body in the year 1975. The authority
shall consist of not more than 14 members, including its chairperson of whom not more than eight shall be
full time members to be appointed by the Central Government.
x The CEA is headed by a chairperson who, as the Chief Executive of the authority, oversees largely the
development of power sector in the country.
x The Central Government shall, from time to time, prepare the National Electricity Policy and tariff policy,
in consultation with the State Governments and the Authority for development of the power system based
on optimal utilisation of resources such as coal, natural gas, nuclear substances or materials, hydro and
renewable sources of energy. Hence statement 3 is not correct.
o It advises the government on matters relating to the National Electricity Policy (NEP) and formulates
short-term and perspective plans for the development of electricity systems.
o The Authority shall prepare a National Electricity Plan in accordance with the National Electricity
Policy and notify such plan once in five years.
x It is the designated authority for cross border trade of electricity. Hence statement 2 is correct.
x It also prescribes the standards on matters such as construction of electrical plants, electric lines and
connectivity to the grid, safety and grid standards and installation and operation of meters.
x It is also responsible for concurrence of hydro power development schemes of central, state and private
sectors for efficient development of river and its tributaries for power generation.
Q 85.C
x Recently, the Centre has prepared a National Mission document to achieve 100 million tonne (MT) coal
gasification by 2030. Hence statement 1 is not correct.
o Concession of 20% on revenue share: For encouraging use of clean sources of fuel, the government
has provided for a concession of 20% on revenue share of coal used for gasification.
o All coal companies have been advised to appoint a nodal officer and to prepare an action plan for
gasifying at least 10% of their coal production. Hence statement 3 is not correct.
x Coal gasification is the process of producing syngas: a mixture consisting primarily of carbon monoxide
(CO), hydrogen (H2), carbon dioxide (CO2), methane (CH4), and water vapour (H2O)—from coal and
water, air and/or oxygen.
x Rather than burning coal directly, gasification converts all of the carbon of the coal into electricity,
hydrogen, and other forms of energy through partial oxidation.
o The hydrogen obtained from coal gasification can be used for various purposes such as making
ammonia. Hence statement 2 is correct.
 34 www.visionias.in ©Vision IAS
.


Q 86.D
x The Mines and Minerals (Development and Regulation) Act, 1957 empowers the central government to
reserve any mine as a captive mine which is utilised for a specific purpose only.
x The Mines and Minerals (Development and Regulation) Amendment Act, 2021 removes the
distinction between captive and non-captive mines. The mines will not be limited to just a specific
purpose/industry/sector. Hence, statement 1 is not correct.
x MMDR Amendment Act, 2021 allows the leaseholders of captive mines to sell up to 50% of their ore
into the open market. Further, it provides that all clearances and licenses granted shall continue till the
reserves have been mined and post the expiry or termination of the lease, will be transferred to the next
successful bidder.
x District Mineral Foundation (DMF) is set up as a non-profit body in the districts affected by the mining
works, to work for the benefit of affected people and areas. The central government can also direct how
the money should be spent from DMF for the development of the area. Hence, statement 2 is not
correct.

Q 87.D
x In line with Atmanirbhar Bharat, the Ministry of Chemicals and Fertilizers recently created the
Scheme for "Promotion of Medical Device Parks" to help the medical device industry. The scheme's
entire financial investment is Rs 400 crore, and its duration is from FY 2020-2021 to FY 2024-2025.
Hence statement 1 is correct.
x How much is the assistance with finances?
o The scheme's entire financial investment is Rs 400 crore. 70 percent of the project cost of shared
infrastructural facilities would be covered by financial support to a selected Medical Device Park.
Financial aid would be 90% of the project cost in the case of the North-Eastern States and the Hilly
States.
o The maximum amount of support available under the scheme for a single Medical Device Park
will be Rs. 100 crores. Hence statement 2 is correct.
o The parks in Himachal Pradesh, Tamil Nadu, Madhya Pradesh, and Uttar Pradesh have received
preliminary approval from the Centre.
x Related Initiatives: In June 2021, the Quality Council of India (QCI) and the Association of Indian
Manufacturers of Medical Devices (AiMeD) launched the Indian Certification of Medical Devices
(ICMED) 13485 Plus scheme to undertake verification of the quality, safety and efficacy of medical devices.
x The Government of India recognised medical devices as a sunrise sector under the ‘Make in India’
campaign in 2014. Hence statement 3 is correct.

 35 www.visionias.in ©Vision IAS


.


Q 88.A
x The Ministry of Women and Child Development is implementing the Swadhar Greh scheme to
provide temporary support and rehabilitation to women who are in difficult circumstances, such as
destitute women, widows, women prisoners released from jail, women victims of natural calamities,
trafficked women/girls rescued, and women affected by domestic violence. Under the scheme, Swadhar
Greh will be set up in every district, with a capacity of 30 women.
x The scheme aims to provide basic needs such as food, shelter, clothing, and medical aid to women in
distress. It also aims to provide emotional support to these women, enabling them to regain their
confidence and emotional strength. The scheme provides various facilities to women, such as counselling,
legal aid, skill development, health and hygiene awareness, and education. Hence statements 1 and 2 are
correct.
x The scheme provides temporary shelter to women for a maximum period of three years. During this period,
the women are provided with various opportunities for rehabilitation and reintegration into mainstream
society. The scheme also provides vocational training and employment opportunities to these women to
help them become financially independent.
x In addition, the Swadhar Greh scheme also provides financial assistance to State Governments and NGOs
for the establishment of such institutions. However, it does not provide financial assistance to NGOs for
carrying out activities related to preventing and combating domestic violence. Hence statement 3 is not
correct.

Q 89.D
x The National Trust under the Ministry of Social Justice & Empowerment in collaboration with key
partners organised the conference “Inclusive India Initiative: Towards an Inclusive India” for intellectual
and developmental disabilities (IDDs) here today. Hence, statement 2 is not correct.
x The ‘Inclusive India Initiative’ of National Trust is specifically catering to persons with intellectual and
developmental disabilities; with an objective to include these people in the mainstream and in all important
aspects of social life, namely education, employment and community ‘Inclusive India’ is about changing
the attitudes. Hence, statement 1 is not correct.

Q 90.C
x The English raised the Territorial Army in 1920 through the Indian Territorial Act of 1920 and comprised
of two wings namely – ‘The Auxiliary Force’ for Europeans & Anglo-Indians, and ‘The Indian Territorial
Force’ for Indian Volunteers.
x After Independence the Territorial Army Act was passed in 1948 and the first Indian Governor
General Shri C Rajagopalachari formally inaugurated the Territorial Army on 09 October 1949 and
this day is celebrated as the TA Day every year. Hence statement 1 is correct.
x It is a voluntary, part-time citizen's army, consisting of persons who are not professional soldiers but
civilians eager to do their bit towards country's defence. Hence statement 2 is correct.
x Its present role is to relieve the regular army from static duties and assist civil administration in dealing
with natural calamities and maintenance of essential services in situations where life of the
communities is affected or the security of the Country is threatened and to provide units for Regular Army
as and when required. Hence statement 3 is correct.
 36 www.visionias.in ©Vision IAS
.
 Q 91.D
x Mission for Integrated Development of Horticulture (MIDH) is a Centrally Sponsored Scheme for the
holistic growth of the horticulture sector covering fruits, vegetables, root & tuber crops, mushrooms, spices,
 flowers, aromatic plants, coconut, cashew, cocoa and bamboo.
o Under MIDH, Government of India (GOI) contributes 60%, of total outlay for developmental
programmes in all the states except states in North East and Himalayas, 40% share is contributed by
State Governments. In the case of North Eastern States and Himalayan States, GOI contributes 90%.
x MIDH Sub-Schemes:
o National Horticulture Mission (NHM) is one of the sub schemes of Mission for Integrated
Development of Horticulture (MIDH) which is being implemented by State Horticulture Missions
(SHM) in selected districts of 18 States and 6 Union Territories.
o Horticulture Mission for North East & Himalayan States (HMNEH): It is being implemented for
overall development of Horticulture in North East and Himalayan states.
o National Horticulture Board (NHB): It is implementing various schemes under MIDH in all States
and UTs.
o Coconut Development Board (CDB): CDB is implementing various schemes under MIDH in all
Coconut growing states in the country.
o Central Institute for Horticulture (CIH): CIH was established at Medi Zip Hima, Nagaland in 2006-
07 for providing technical backstopping through capacity building and training of farmers and Field
functionaries in the North Eastern Region.
x Hence option (d) is the correct answer.

Q 92.A
x India is bestowed with large resources of iron ore. Iron ore occurs in different geological formations. The
major economic deposits of hematite iron ore are found associated with the volcano-sedimentary
Banded Iron Formation (BIF) of the Precambrian age. Hence, statement 1 is correct.
x Around 60 percent of Hematite ore deposits are found in eastern parts of India, whereas, 92 percent of
magnetite ore deposits are found in southern parts of India (especially Karnataka). Hence, statement 2
is not correct.
x Of magnetite and hematite, hematite is considered to be superior because of its high-grade quality and
lumpy nature, which is consumed by a large number of steel and sponge iron industries in the country.
Hence, statement 3 is not correct.

Q 93.A
x India launched its Antarctic expeditions in 1981. The programme has now built three permanent research
base stations - namely Dakshin Gangotri, Maitri and Bharati. The National Centre for Polar and
Ocean Research (NCPOR), Goa, manages the entire Indian Antarctic programme.
x Dakshin Gangotri was the first scientific base station of India situated in Antarctica. It is located at a
distance of 2,500 kilometres from the South Pole. It is currently being used as a supply base and transit
camp.
x Maitri and Bharati were respectively the second and third research stations in Antartic region. They
research the age of the Antarctic ice, and the speed of it's melting.
x Himadri is India’s research station located at the International Arctic Research base in Norway. The
research station Himadri carries out studies in disciplines like Glaciology, Atmospheric sciences &
Biological sciences, etc.
x Hence, option (a) is the correct answer.

Q 94.D
x Lying entirely in the northern hemisphere, mainland India extends between latitudes 8°4' and 37°6'
north, longitudes 68°7' and 97°25' east, and measures about 3,214 km from north to south between the
extreme latitudes and about 2,933 km from east to west between the extreme longitudes. Hence statement
1 is correct.
x It has a land frontier of about 15,200 km. Hence statement 2 is not correct.
x The total length of the coastline of the mainland, Lakshadweep Islands, Andaman, and Nicobar
Islands is 7,516.6 km. Hence statement 3 is correct.

 37 www.visionias.in ©Vision IAS


.
 Q 95.C
x About NIPUN Bharat or National Initiative for Proficiency in Reading with Understanding and
Numeracy: NIPUN Bharat has been launched under the Centrally Sponsored Scheme ‘Samagra
 Shiksha’. Hence statement 1 is correct.
x Vision: To create an enabling environment to ensure the universal acquisition of foundational literacy and
numeracy. So that by 2026-27 every child achieves the desired learning competencies in reading,
writing and numeracy at the end of Grade III and not later than Grade V. Hence statement 2 is not
correct.
x Implementing agency: Department of School Education and Literacy, Ministry of Education.
x Beneficiaries: Children in the age group of 3 to 9 years including preschool to grade 3 & children who are
in class4 and 5 and have not attained the foundational skills.
x Implementation strategy: A five-tier Implementation mechanism will be set up at the National-State-
District-Block and School level. Hence statement 3 is not correct.

Q 96.B
x Central Board of Indirect Taxes & Customs (CBIC):
o The Central Board of Indirect Taxes is under the Department of Revenue, Finance Ministry,
Government of India.
o The CBIC was previously known as the Central Board of Excise and Customs (CBEC).
o It engages in policy formulation of matters concerned with the levy and collection of central excise
duties, customs, central goods, and services tax and IGST, administration of these taxes, prevention of
smuggling, and also narcotics to the extent prescribed.
o The CBIC is the administrative authority for its subordinate organizations, including Central Excise and
Central GST Commissioners, Custom Houses, and the Central Revenues Control Laboratory.
o CBIC is a statutory body established under the Central Boards of Revenue Act, of 1963.
o CBIC was formed in 1964 when the Central Board of Revenue was split into the Central Board of Direct
Taxes (CBDT) and the Central Board of Excise and Customs, which was renamed the Central Board of
Indirect Taxes and Customs in 2018.
o The Central Board of Revenue was set up under the Central Board of Revenue Act, 1924 as the apex
body of the Income Tax Department.
x Central Board of Direct Taxes (CBDT):
o It is a statutory body established as per the Central Board of Revenue Act, of 1963. Hence, statement
1 is not correct.
o It is India’s official financial action task force unit.
o It is administered by the Department of Revenue under the Ministry of Finance. Hence, statement
2 is correct.
o To note that originally there was a board called the Central Board of Revenue that functioned as the
apex body of the Income Tax Department. The said board was set up under the Central Board of
Revenue Act, of 1924, and was in charge of both direct and indirect taxes. The Central Board of
Revenue got split in 1964 into two boards:
9 Central Board of Direct Taxes
9 Central Board of Excise and Customs

Q 97.C
x About National Credit Framework (NCrF): Proposed as part of National Education Policy (NEP)
2020, is an umbrella framework for skilling, re-skilling, up-skilling, accreditation and evaluation in
education and skilling institutions and the workforce. Hence statement 2 is correct.
x Jointly developed by a High-Level Committee with members from UGC, AICTE, CBSE, NCERT, etc.
x It aims to seek to integrate the credits earned through school, higher and vocational & skill education to
ensure flexibility and mobility among them. It will act as guidelines to be followed by schools, colleges and
universities in adopting the credit system.
x NCrF will enable learners to earn credits not just through classroom learning but through co-
curricular, extracurriculars, recognition of prior learning, etc. .
x NCrF would be only one credit framework for higher education, school education and skill education.
Hence statement 1 is correct.
x According to the framework, an academic year will be defined by the number of hours a student puts
in. Credits will be provided to them accordingly at the end of each academic year.

 38 www.visionias.in ©Vision IAS


.


Q 98.B
x The “Swachhta Udyami Yojana” is for extending financial assistance for the Construction, Operation &
Maintenance of Pay and Use Community Toilets in Public Private Partnership (PPP) Mode and Procurement
& Operation of Sanitation related Vehicles. Hence, statement 2 is correct.
x The Scheme has been launched on the 2nd October 2014, the birth anniversary of Mahatma Gandhi by
the Hon’ble Minister of State for Social Justice & Empowerment. Hence, statement 3 is not correct.
x This Scheme has twin objectives of cleanliness, providing livelihood to Safai Karamcharis and
liberating Manual Scavengers to achieve the overall goal of “Swachh Bharat Abhiyan” initiated by the
Hon’ble Prime Minister. Hence, statement 1 is not correct.
x Entrepreneurs among safari karmacharis and identified manual scavengers can avail of loans up to the
defined ceiling at a concessional rate of interest @ 4% per annum.
x In the case of women beneficiaries, there is a rebate of 1% in the rate of interest charged. Hence,
statement 4 is not correct.

Q 99.C
x An Agriculture Accelerator Fund will be set up to encourage agri-startups by young entrepreneurs
in rural areas. The fund will aim at bringing innovative and affordable solutions for challenges faced by
farmers. It will also bring in modern technologies to transform agricultural practices, and increase
productivity and profitability. Hence option (c) is the correct answer.
x Digital public infrastructure for agriculture will be built as an open-source, open-standard and interoperable
platform for public good. This will enable inclusive, farmer-centric solutions through relevant information
services for crop planning and health; improved access to farm inputs, credit, and insurance; help for crop
estimation and market intelligence. It will support the growth of the agri-tech industry and start-ups
operating in the space.
x The proposed Agriculture Accelerator Fund can provide a supportive environment that can help agritech
startups grow faster by providing them with resources, exposure, and opportunities to succeed.

Q 100.C
x Recently, the Cabinet Committee on Economic Affairs approved the scheme on Green Energy
Corridor (GEC) Phase-II for Intra-State Transmission System (InSTS).
x In year 2012, a study was conducted by Power Grid Corporation of India Limited (PGCIL) wherein it was
found that power evacuation and transmission infrastructure in near vicinities of potential sites was less and
therefore, dedicated transmission infrastructure for large scale solar and wind power plants was planned.
GEC comprises of both Inter State Transmission System (ISTS) and Intra State Transmission System
(InSTS) along with the setting up of Renewable Energy Management Centre (REMC) and the control
infrastructure like, reactive compensation, storage systems, etc.

 39 www.visionias.in ©Vision IAS


.
 x Objectives of GEC:
o It aims at synchronizing the electricity produced from renewable resources, such as wind and solar,
with the conventional power stations in the grid. Hence statement 1 is correct.
 o It aims to achieve the target of 450 GW installed RE capacity by 2030. Hence statement 2 is correct.
o The objective of the GEC is to evacuate approx. 20,000 MW of large-scale renewable power and
improvement of the grid in implementing states.
x GEC-1:
o Phase 1 of the Green Energy Corridor is already under implementation in 8 RE rich states i.e. Gujarat,
Andhra Pradesh, Karnataka, Himachal Pradesh, Maharashtra, Madhya Pradesh, Tamil Nadu, and
Rajasthan.
o It is working for the grid integration and power evacuation of about 24GW of Renewable Energy.
x GEC-2:
o It will facilitate grid integration and power evacuation of approximately 20 GW of Renewable Energy
(RE) power projects in seven States namely, Gujarat, Himachal Pradesh, Karnataka, Kerala, Rajasthan,
Tamil Nadu and Uttar Pradesh.
o The transmission systems will be created over a period of five year from Financial Year 2021-22 to
2025-26.
o It is targeted to be set up with a total estimated cost of Rs. 12, 031 crores, and the Central Finance
Assistance (CFA) will be 33% of the project cost.
9 The CFA will help in offsetting the Intra-State transmission charges and thus keep the power costs
down.

You might also like